dfshgdfgbcfbxf

Réussis tes devoirs et examens dès maintenant avec Quizwiz!

A patient is lying supine with hips and knees extended and hands behind the head. The patient is able to raise the head, shoulders, and thorax from the treatment table but is unable to come to a complete long-sitting position. What muscle should the physical therapist target for a strengthening program? 1. Iliopsoas 2. External abdominal oblique 3. Quadratus lumborum 4. Upper rectus abdominis

1 Iliopsoas 1. The abdominal muscles are active during a sit-up (with the knees extended) until the spine is completely flexed (head, shoulders, thorax lifted from surface). In order to come to a long-sitting position, however, the hips must be flexed, and the abdominals cannot perform this action because they do not cross the hip joint. Therefore, the hip flexors (iliopsoas among others) would have to complete this motion. The inability to achieve a long-sitting position would suggest weakness in the iliopsoas muscle.

A patient has an ulcer on the right lateral malleolus. The patient's resting heart rate is 72 bpm, respiratory rate is 15 breaths/minute, right brachial blood pressure is 120/80 mm Hg, and right posterior tibial blood pressure is 70/50 mm Hg. Which of the following values MOST accurately represents the patient's ankle-brachial index? 1. 0.58 2. 0.63 3. 0.97 4. 1.71

1. 0.58 The ankle-brachial index is the highest ankle systolic pressure divided by the highest brachial systolic pressure. A value of 0.58 correctly represents the ratio of the ankle systolic pressure divided by the brachial systolic pressure for the data provided.

During examination of a patient, a physical therapist notes that one of the patient's feet is erythematous, swollen, and significantly warmer to the touch than the contralateral foot. The patient denies any pain or trauma. The patient has a history of diabetes, peripheral vascular disease, and chronic heart failure. The findings are MOST consistent with the presence of which of the following conditions? 1. Grade III sprain of the talocrural ligament 2. Fractured distal tibia 3. Bone cancer 4. Charcot disease

1. A patient with a sprain would have a history of trauma. In addition, the patient would have referred or local pain. 2. A patient with a fracture would have a history of trauma. In addition, the patient would have referred or local pain. 3. A patient who has bone cancer would likely have referred or local pain. Other warning signs, such as unexplained weight loss and night sweats, would be expected. 4. The signs and symptoms, along with a history of diabetes, are most consistent with Charcot disease.

During the gait evaluation of a patient who has a transfemoral prosthesis, a physical therapist notices that the patient laterally bends excessively toward the prosthetic side during midstance phase. Which of the following factors is MOST likely to cause this gait deviation? 1. A prosthesis that is too short 2. A prosthetic socket that is too small 3. Inadequate prosthesis suspension 4. A locked knee unit

1. A prosthesis that is too short 1. A prosthesis that is too short causes a patient to laterally bend toward the prosthetic side during stance phase (pp. 679-680). 2. A prosthetic socket that is too small would likely functionally lengthen the prosthetic side and also cause vaulting (p. 681). 3. Inadequate suspension causes vaulting (p. 681). 4. Inadequate knee flexion causes vaulting (p. 681).

A physical therapist observes that a patient with end-stage renal disease has significant exercise intolerance. Which of the following factors is the MOST likely cause of the exercise intolerance? 1. Anemia 2. Hypotension 3. Hypertriglyceridemia 4. Increased VO2

1. Anemia 1. Persons with end-stage renal disease have limitations to exercise due to reduced oxygen-carrying capacity associated with anemia. 2. End-stage renal disease is associated with hypertension, not hypotension. 3. Hypertriglyceridemia does not directly alter the exercise response. It has been linked to cardiovascular disease, but by itself would not be the most likely cause for exercise intolerance for a person who has end-stage renal disease. 4. End-stage renal disease is associated with decreased VO2.

A physical therapist examines a patient who has peripheral neuropathy. Muscle strength testing results in the following findings: Muscle Right Left Biceps femoris Good (4/5) Good (4/5) Quadriceps Good (4/5) Good (4/5) Anterior tibialis Poor (2/5) Good (2/5) Gastrocnemius Poor (2/5) Good (2/5) Which of the following orthoses is MOST appropriate for this patient? 1. Ankle-foot 2. Functional foot 3. Knee-ankle-foot 4. Hip-knee-ankle-foot

1. Ankle-foot 1. With polyneuropathy associated with type 2 diabetes, there is decreased sensation as well as muscle strength deficit. The ankle-foot orthoses provide needed ankle control. (p. 224) 2. Functional foot orthoses would not address the need for ankle control (p. 198). 3. Quadriceps strength is adequate to provide knee control (p. 239-240). 4. Quadriceps strength is adequate to provide hip and knee control (pp. 252-253).

A patient who is being taught a new motor task is MOST likely to require full attention to the task and extrinsic feedback during which phase of motor learning? 1. Cognitive 2. Associative 3. Autonomous 4. Expert

1. Cognitive Learning in the first phase of motor learning (the cognitive phase) requires full attention to the task and extrinsic feedback

A patient with a mild closed head injury and bilateral femur fractures requires instruction in a lower extremity exercise program. To plan the most effective teaching methods for this patient, what is MOST critical to assess at the INITIAL visit? 1. Comprehension of written, verbal, and demonstrated instructions 2. Short-term memory capacity 3. Auditory and visual status 4. Any personality changes compared to the patient's premorbid status

1. Comprehension of written, verbal, and demonstrated instructions 1. The most critical component to assess is comprehension, to allow appropriate planning in instructional strategies. To effectively communicate with patients, a physical therapist must understand the patients' comprehension of what is being said, in addition to the patient's ability to read materials given to them and their understanding of the value of the therapist's recommendations within the context of their lives. 2. If the patient does not initially understand the information, then having short-term memory is irrelevant. 3. Although auditory and visual status would require modification to teaching style, the most critical component is assessing the patient's ability to comprehend the information that is presented. 4. Personality changes would be irrelevant to the scenario.

To manually assess a patient's lower extremity circulation, a physical therapist should palpate the patient's peripheral pulse at which of the following locations? 1. Dorsal foot, near the base of the first metatarsal 2. Lateral lower leg, just posterior to the fibular head 3. Lateral ankle, just inferior to the lateral malleolus 4. Plantar foot, just medial to the medial calcaneal tuberosity

1. Dorsal foot, near the base of the first metatarsal 1. The therapist should palpate the dorsal pedal pulse, which is found on the dorsal aspect of the foot near the base of the first metatarsal. The dorsalis pedis pulse is often used to assess a patient's circulation because of its distal location on the lower extremity. (pp. 1101-1102) 2. The dorsalis pedis pulse is preferred for assessment of lower extremity circulation due to its distal location. The popliteal pulse may also be palpated, but it is located posterior to the knee, not in the lateral leg. (p. 979) 3. The dorsalis pedis pulse is preferred for assessment of lower extremity circulation due to its distal location. The posterior tibial pulse may also be palpated, but it is located at the medial ankle just posterior to the medial malleolus, not the lateral ankle. (p. 1102) 4. The dorsalis pedis pulse is preferred for assessment of lower extremity circulation due to its distal location. The pulse would be palpated on the dorsal surface of the foot, not the plantar surface. The plantar foot does not possess a pulse site used in lower extremity circulation assessment. (p. 1102)

A 50-year-old male patient reports a sudden onset of severe right great toe pain that began the morning of the current day. He reports being very tired and having chills over the past 24 hours. The patient has a 102°F (38.8°C) temperature, and his right great toe is warm and swollen. What is the MOST likely cause of these symptoms? 1. Gout 2. Bunion 3. Turf toe 4. Rheumatoid arthritis

1. Gout 1. Gout is the most common form of inflammatory arthritis in men older than 40 years of age. The onset is usually sudden, often during the night or early morning. Classic findings are warmth, swelling, and severe pain in the first metatarsophalangeal joint

A physical therapist is taking the history of a patient with low back pain. If the therapist suspects the pain is caused by an inflammatory reaction, which of the following questions is BEST for the therapist to ask? 1. Is your pain constant or intermittent? 2. What activity bothers you the most? 3. Is it difficult to stand up straight after you've been sitting? 4. Does your pain radiate down into the leg?

1. Is your pain constant or intermittent? Constant pain is systemic, and mechanical pain generally changes with positions or activities. Asking the patient to report if the pain is constant or intermittent would be best for differentiating between systemic and mechanical pain.

When examining a patient with a history of alcohol abuse, a physical therapist notes that the patient demonstrates fine resting tremors and hyperactive reflexes. The patient reports frequent right upper quadrant pain. Which of the following additional signs is MOST likely to be present? 1. Jaundice 2. Hyperhidrosis 3. Hypotension 4. Nocturnal cough

1. Jaundice 1. With a history of alcohol abuse and the presence of fine resting tremors and right upper quadrant pain, the patient is presenting a history and signs and symptoms consistent with liver disease. Jaundice is a skin change associated with disease of the hepatic system. (Goodman, p. 363) 2. Hyperhidrosis, or excessive sweating, can be present with several conditions, such as endocrine disorders, but is not associated with liver disease (Taber's, p. 1164). 3. Hypotension is not listed as a sign of liver disorders (Goodman, p. 363). 4. A nocturnal cough can be associated with rheumatic fever, but is not characteristic of liver disease (Goodman, p. 260).

During the initial evaluation of a patient with right upper extremity pain, the physical therapist notes that the patient's right scapula is significantly lower than the left scapula. Shortness of which of the following muscles on the right is MOST likely to lead to this patient's postural presentation? 1. Latissimus dorsi 2. Upper trapezius 3. Levator scapulae 4. Teres major

1. Latissimus dorsi 1. The latissimus dorsi depresses the shoulder girdle and can be short in patients with depressed scapulae. 2. The upper trapezius, if short, would elevate the scapula. 3. The levator scapulae, if short, would elevate and downwardly rotate the scapula. 4. The teres major, if short, would adduct, extend, and medially (internally) rotate the glenohumeral joint.

When a patient makes a fist, the head of the third metacarpal is level with the second and fourth metacarpals. This finding is indicative of which of the following conditions? 1. Lunate dislocation 2. Capitate instability 3. Scaphoid subluxation 4. Lunotriquetral ligament tear

1. Lunate dislocation 1. This observation describes the Murphy sign, which indicates lunate dislocation. Normally, the third metacarpal should project distally to the second and fourth metacarpals. 2. Capitate instability is indicated by symptom reproduction, apprehension, or pain during dorsal capitate displacement testing. 3. Scaphoid subluxation is indicated by a positive result on the Watson (scaphoid shift) test. 4. A lunotriquetral ligament tear is indicated by pain, crepitus, or abnormal movement during the lunotriquetral shear test.

Which of the following statements MOST accurately describes the proper testing technique when using monofilaments? 1. Place the monofilament perpendicular to the skin surface, bending the monofilament. 2. Place the monofilament perpendicular to the skin surface without bending the monofilament. 3. Place the monofilament parallel to the skin surface and move it back and forth. 4. Place the monofilament parallel to the skin surface and move it up and down.

1. Place the monofilament perpendicular to the skin surface, bending the monofilament. The correct technique for Semmes-Weinstein monofilament sensory testing is to place the monofilament perpendicular to the skin surface and press until the filament bends.

Following an acquired brain injury, a patient has uncoordinated movements and is working on relearning how to carry a cup without spilling any liquid. Which of the following patterns of practice is MOST appropriate for long-term retention of the task and transfer to novel conditions? 1. Provide feedback about the outcome of the performance after a practice session where the patient carried cups with random levels of liquid. 2. Provide continuous verbal cues about the pattern of movement during the practice session as the patient carries cups with random levels of liquid. 3. Provide feedback about the results of the performance after a practice session where the patient carried cups with progressively more liquid. 4. Provide continuous manual cues to improve the pattern of movement during the practice session as the patient carries cups with progressively more liquid.

1. Provide feedback about the outcome of the performance after a practice session where the patient carried cups with random levels of liquid. 1. Knowledge of results and a random practice pattern improve long-term retention and task transfer to novel conditions.

Manual muscle testing of a patient's pelvic floor muscles reveals a grade of Poor (2/5). Which of the following positions is BEST to begin strengthening? 1. Supine 2. Standing 3. Seated 4. Walking

1. Supine A grade of Poor (2/5) is defined as full excursion in a gravity-eliminated position. Supine position would be a gravity-eliminated position to begin strength training.

A patient is performing a Phase I (inpatient) cardiac rehabilitation exercise session. The physical therapist should terminate low-level activity if which of the following changes occurs? 1. The diastolic blood pressure increases to 115 mm Hg. 2. The respiratory rate increases to 20 breaths/minute. 3. The systolic blood pressure increases to 175 mm Hg. 4. The heart rate increases by 18 bpm.

1. The diastolic blood pressure increases to 115 mm Hg. 1. During Phase I (inpatient) cardiac rehabilitation, vital sign parameters with activity that warrant termination are: diastolic blood pressure of 110 mm Hg or greater, systolic blood pressure above 210 mm Hg or an increase greater than 20 mm Hg from resting, and a heart rate that increases beyond 20 bpm above resting.

A 90-year-old patient with chronic heart failure has been nonambulatory and has resided in a nursing home for the past year. The patient was recently admitted to the hospital after an episode of dehydration. Which of the following plans for prophylactic respiratory care is MOST appropriate? 1. Turning, coughing, and deep breathing every 1 to 2 waking hours 2. Vigorous percussion and vibration 4 times/day 3. Gentle vibration with the foot of the bed elevated 1 time/day 4. Segmental postural drainage using standard positions throughout the day

1. Turning, coughing, and deep breathing every 1 to 2 waking hours 1. A patient who is immobile and restricted to bed is at risk for developing atelectasis, or partial collapse of lung tissue, which can then lead to pneumonia. Frequent position changes with deep breathing and coughing will help prevent development of atelectasis. 2. Given that this patient is elderly and does not have a diagnosis of secretion retention, vigorous percussion and vibration is not indicated. 3. Vibration with the head down will not be tolerated in this elderly patient with chronic heart failure. 4. Standard postural drainage positions will involve the patient's head being below the level of the heart. This will not be tolerated in this elderly patient with chronic heart failure.

A patient who has low back pain has been undergoing treatment for two sessions. During the current treatment session, the patient tells the physical therapist that the pain is centralizing with the extension exercises but is as intense as it was at the first treatment session. The patient is frustrated by this reaction. The therapist's MOST appropriate response is to: 1. continue with the present program. 2. eliminate the extension exercises. 3. consult the patient's physician about the situation. 4. progress to trunk flexion exercises.

1. continue with the present program. 1. With an extension exercise program, centralization indicates that the patient is improving, even though the pain may intensify. Therefore, the therapist should continue with the extension exercise program.

During steady rate exercise, the blood pressure in a healthy person MOST typically responds with: 1. systolic pressure increase and no change in diastolic pressure. 2. no change in systolic pressure and marked decrease in diastolic pressure. 3. systolic pressure decrease and diastolic pressure increase. 4. no change in systolic pressure until the end point of the exercise session.

1. systolic pressure increase and no change in diastolic pressure. Steady rate exercise, such as jogging or cycling, causes a relatively rapid increase in systolic pressure with diastolic pressure remaining the same or decreasing only slightly. Systolic pressure may level off somewhat during the exercise if exercise intensity remains the same.

Cool-down exercises are performed immediately after a general aerobic exercise program PRIMARILY to prevent: 1. venous pooling. 2. cardiac arrhythmia. 3. decreased body temperature. 4. muscle tightening.

1. venous pooling. 1. During aerobic exercise, vascular dilation occurs in order to optimize blood flow and the venous system relies on muscular pumping in order to return blood back to the heart. Without a cool-down period, there would be no muscular pumping and, therefore, blood would pool in the venous system. 2. Cardiac arrhythmia may occur during exercise but is not specifically addressed with cool-down exercises. 3. Cessation of activity by itself would result in a gradual decrease in body temperature, and specific cool-down exercises are not necessary. 4. Muscle tightening may occur as a result of metabolites accumulating in the circulatory system, but this would occur as a result of the venous pooling.

A physical therapist is testing the deep tendon reflex of a patient as shown in the photograph. The patient has hypothyroidism. Which of the following reflex grades is the therapist MOST likely to find in the patient? 1. 0 2. 1+ 3. 2+ 4. 3+

2. 1+ 1. A response of 0 represents areflexia, which is not expected in a patient who has hypothyroidism. Areflexia is associated with lower motor neuron disorders. 2. Hyporeflexia (1+) is associated with hypothyroidism. 3. A 2+ response represents a normal deep tendon reflex, which is not expected in a patient who has hypothyroidism. 4. A 3+ response represents a hyperreflexive response, which is not expected in a patient who has hypothyroidism. Hyperreflexia is associated with upper motor neuron disorders.

An initial physical therapy evaluation is conducted for an older adult patient 1 day after the patient had a left total hip arthroplasty (noncemented) using a posterolateral approach. The patient has no complicating medical history and was active and independent preoperatively. Which of the following activities is LEAST appropriate for the first week of therapy? 1. Active-assistive positioning of the left hip to 60° of flexion 2. Active left hip abduction in right sidelying position 3. Independent bed mobility with the use of a trapeze 4. Walking with moderate assistance with a standard walker to 25 ft (7.6 m)

2. Active left hip abduction in right sidelying position 1. Patients are taught to avoid excessive hip flexion, usually beyond 80°. 2. Although protocols depend on the surgeon and the approach, it is generally recommended that against-gravity hip abduction exercises not begin until 5 to 6 weeks after surgery. 3. Bed mobility would be started 1 or 2 days postoperatively. 4. Gait training would be started 1 or 2 days postoperativel

Following acute knee trauma, which of the following examination findings indicates the need for referral for radiographic imaging? 1. Tenderness to palpation of the tibial tuberosity 2. Active range of motion knee flexion limited to 75° 3. Tenderness to palpation of the medial femoral condyle 4. Full weight-bearing gait tolerance limited to 20 ft (6.1 m)

2. Active range of motion knee flexion limited to 75° 1. Tenderness to palpation at the fibular head or patella, not the tibial tuberosity, is an indication for radiographs. 2. According to the Ottawa knee rules, a patient who has post acute knee trauma should be referred for radiographs if any of the following five criteria are present: age 55 years or older, tenderness at the fibular head or patella, inability to flex the knee greater than 90°, or inability to weight-bear for four steps. 3. Tenderness at the fibular head or patella, not medial femoral condyle, is an indication for radiographs. 4. Inability to take 4 steps, not walking 20 ft (6.1 m), is an indication for radiographs.

A patient with sudden onset of weakness of the facial muscles on the right side is unable to wrinkle the forehead, smile, pucker the lips, or wrinkle the nose. There is an absence of tearing in the patient's right eye and dryness of the mouth. The patient's corneal reflex is absent on the right but normal on the left, and pinprick and temperature sensation are normal on both sides of the face. This presentation is characteristic of: 1. trigeminal neuralgia. 2. Bell palsy. 3. left cortical cerebrovascular accident. 4. oculomotor nerve (CN III) damage.

2. Bell palsy. 2. All of the signs and symptoms in the question are indicative of a partial nerve lesion affecting the facial nerve (CN VII). This type of lesion is Bell palsy. (Schenkman, p. 320)

A patient with a cerebellar stroke has received functional balance training for 4 weeks. Which of the following tests is MOST appropriate to measure the effectiveness of the physical therapy intervention? 1. Romberg Test 2. Berg Balance Scale 3. Fugl-Meyer Assessment 4. Barthel Index

2. Berg Balance Scale The Berg Balance Scale is an objective measure of static and dynamic balance abilities and consists of 14 commonly performed functional tasks; therefore, it is the most appropriate tool to use as a measure of intervention effectiveness.

A patient with a chronic stroke walks with a hemiparetic gait pattern at a velocity of 1.31 ft (0.4 m)/second. Which of the following interventions will MOST improve the patient's gait speed? 1. Practicing individual components of walking 2. Body-weight supported gait training at fast speeds 3. Body-weight supported gait training at slow speeds 4. Hip flexor and plantar flexor muscle strength training

2. Body-weight supported gait training at fast speeds 1. Practicing components of walking in isolation is not as effective as practicing the entire task of gait for improving gait speed. 2. Studies support the effectiveness of body-weight supported gait training for improving the gait speed of individuals who have chronic stroke. Additionally, practicing at a fast speed produces greater gains in gait speed, compared to practicing at slower speeds. 3. Practicing at a fast speed produces greater gains in gait speed, compared to practicing at slower speeds. 4. Improving muscle strength and power by resistance training is not associated with improved gait speeds in patients who have chronic stroke.

Upon removal of a wet-to-dry dressing from a patient's draining wound, a physical therapist observes that the skin immediately surrounding the wound is macerated. What should the therapist recommend for future wound care? 1. Continue using the current dressing type. 2. Change to a more absorbent dressing. 3. Leave the dressing on longer between dressing changes. 4. Change to a pressure-type dressing.

2. Change to a more absorbent dressing. 1. If a wound develops periwound maceration, healing will be impaired (p. 27). The dressing type should be changed to prevent maceration. (p. 140) 2. If a wound is too moist, periwound tissue might become macerated. Macerated skin is more fragile than healthy skin (pp. 27-28). A draining wound should be covered with a dressing that absorbs moisture well enough to protect periwound tissue from maceration. (p. 140) 3. Maceration indicates that the dressing is not absorbent enough to control the wound drainage. Leaving the current dressing on for longer periods of time will worsen the maceration. The dressing type should be changed, or the current dressing should be changed more frequently. (p. 140) 4. If skin is macerated, the dressing should be changed to a more absorbent type, not one that increases pressure (p. 140).

Clubbing of the fingers is MOST associated with which of the following conditions? 1. Lymphedema 2. Chronic obstructive pulmonary disease 3. Chronic venous insufficiency 4. Complex regional pain syndrome

2. Chronic obstructive pulmonary disease 1. Lymphedema is not commonly associated with digital clubbing. While lymphedema is associated with chronic swelling of the extremities, chronic hypoxia is not commonly observed. 2. Conditions that chronically interfere with tissue perfusion and nutrition may cause clubbing. Pulmonary disease is the most predominant cause of digital clubbing and is present in 75% to 85% of the cases in which clubbing is noted. 3. Chronic venous insufficiency is not commonly associated with digital clubbing. Hemosiderin staining, lower extremity swelling, and stasis ulcers are more characteristic of venous insufficiency. 4. Digital clubbing is not commonly associated with complex regional pain syndrome. Abnormal pain and trophic changes are characteristic of complex regional pain syndrome.

The test shown in the photograph assesses which of the following tracts? 1. Lateral spinothalamic 2. Dorsal column-medial lemniscal 3. Anterior spinothalamic 4. Ventral spinocerebellar

2. Dorsal column-medial lemniscal 2. The photograph depicts two-point discrimination testing. The dorsal column-medial lemniscal tract is responsible for discriminative sensations such as two-point discrimination. (O'Sullivan, pp. 100-101)

For an individual with a T12 spinal cord injury to be able to walk, achievement of which of the following ranges of motion is MOST important? 1. 30° of ankle plantar flexion 2. Full hip extension 3. Normal knee flexion 4. 110° straight-leg raise

2. Full hip extension 1. Ten degrees of dorsiflexion and absence of ankle plantar flexion contractures are required for normal walking. 2. Full hip extension is required for a person who has a T12 level injury to be able to walk. 3. Normal knee flexion is not required for normal walking for a person who has a T12 level injury. 4. Although 110° straight-leg raise is important for long sitting and floor-to-wheelchair transfers, this is not required for gait.

A patient who is 8 months pregnant has an abdominal diastasis recti with a separation of 1.5 inches (3.8 cm). Which of the following exercises would be MOST appropriate INITIALLY for abdominal strengthening in a supine position? 1. Trunk curls 2. Hooklying head lifts 3. Pelvic-tilt leg sliding 4. Bilateral leg lowering

2. Hooklying head lifts 1. Trunk curls are contraindicated for a patient who has diastasis recti (p. 948). 2. Supine hooklying head lifts emphasize the rectus abdominis muscle and are least likely to increase the separation of the diastasis recti (p. 947). 3. Pelvic-tilt leg sliding is more advanced than head lifts and would not be the most appropriate exercise to use initially (p. 947). 4. Bilateral leg-lowering is an advanced abdominal strengthening exercise that causes excessive low back strain and should not be performed during pregnancy (p. 951).

Which of the following nervous system complications would indicate the poorest prognosis for a patient with acquired immunodeficiency syndrome (AIDS)? 1. Toxoplasmosis 2. Leukoencephalopathy 3. Myelopathy 4. Polyneuropathy

2. Leukoencephalopathy 1. Toxoplasmosis is a complication associated with acquired immunodeficiency syndrome but is not a marker of a poor prognosis. 2. Although all of the options are serious complications in patients with acquired immunodeficiency syndrome, multifocal leukoencephalopathy is the most serious, and death occurs in a few months after it is diagnosed. 3. Myelopathy is a common complication of acquired immunodeficiency syndrome but is not indicative of a poor prognosis. 4. Polyneuropathy is a common complication of acquired immunodeficiency syndrome but is not indicative of a poor prognosis.

A patient with a recent onset of rheumatoid arthritis is MOST likely to report which of the following symptoms? 1. Heat intolerance 2. Malaise and fatigue 3. Tension headaches 4. Nausea and diarrhea

2. Malaise and fatigue 1. Heat intolerance is not associated with rheumatoid arthritis. Heat intolerance is more indicative of metabolic and endocrine dysfunction. 2. Malaise and fatigue are common symptoms during the beginning stages of rheumatoid arthritis. 3. Tension headaches are not associated with rheumatoid arthritis. This type of headache is most often caused by stress or anxiety. 4. Nausea and diarrhea are not associated with rheumatoid arthritis. Nausea and diarrhea would be more indicative of gastrointestinal dysfunction or another type of systemic illness.

Which of the following locations corresponds to the sensory function of the nerve root exiting between the third and fourth lumbar vertebrae? 1. Lateral knee joint line 2. Medial knee joint line 3. Plantar aspect of the heel 4. Dorsal web space between the first and second toes

2. Medial knee joint line 1. The lateral knee joint line is supplied by the L5 nerve root, which exits between the fifth lumbar vertebrae and first sacral vertebrae (p. 100). 2. The L3 nerve root exits between the third and fourth lumbar vertebrae and supplies sensory information from the region of the medial knee joint line (p. 95). 3. The plantar aspect of the heel is supplied by the S1 nerve root, which exits below the first sacral vertebrae (p. 99). 4. The dorsal web space between the first and second toes is supplied by the L5 nerve root, which exits between the fifth lumbar vertebrae and first sacral vertebrae

A physical therapist places a patient on a strength training program for the lower extremities. The mode of exercise is a double-leg press unit. After 1 week, the patient shows a 10-lb (4.5-kg) increase in the amount of weight the patient is able to lift. What is the MOST likely cause of the patient's increase in strength? 1. Muscle fiber hypertrophy 2. Neurological adaptation 3. Hyperplasia of the muscle fibers 4. Increase in the amount of actin and myosin

2. Neurological adaptation 2. Strength increase in muscle is due to a number of factors, including neurological adaptation and muscle fiber hypertrophy with an increase in actin and myosin. Long-term changes in muscle strength are due to all of the factors listed. However, short-term changes, such as changes in 1 week, are most likely to due to neurological factors such as more efficient motor unit recruitment, autogenic inhibition, and more efficient coactivation of muscle groups.

A patient who has right hemiplegia is unable to avoid obstacles on the right side of the hallway while walking with a walker. During meals, the patient does not eat foods placed on the right side of the plate. Which of the following tests is MOST appropriate to perform? 1. Patient looks at a fixed target in front while turning the head side to side. 2. Patient detects therapist's fingers from the side while looking straight ahead. 3. Patient shifts gaze from one object to another quickly with the head stationary. 4. Patient tracks therapist's finger moving vertically, horizontally, and diagonally.

2. Patient detects therapist's fingers from the side while looking straight ahead. 1. A test that involves moving the head rapidly while focusing on a fixed target directly in front is used to examine function of the vestibuloocular reflex (VOR). 2. Ignoring foods on one side of a plate and difficulty orienting to stimuli in a specific area of space are symptoms of visual field loss. Disruption of the sensory input provided by the optic nerve (CN II) will result in loss of vision in some or all of a field of view. To examine the visual field, the examiner brings his or her fingers from behind the patient's head at eye level while the patient looks straight ahead. 3. Shifting gaze back and forth quickly between two targets is a test for saccadic eye movement. 4. A test that involves following a moving target in different directions is used to assess pursuits, which stimulate all pairs of eye muscles.

A patient who has rheumatoid arthritis comes to physical therapy with signs of muscle atrophy, ecchymosis, puffy cheeks, and a diagnosis of osteoporosis. Which of the following medications is the patient MOST likely taking? 1. Penicillin (Ampicillin) 2. Prednisone (Deltasone) 3. Acetylsalicylic acid (aspirin) 4. Gold salts

2. Prednisone (Deltasone) 1. The primary side effect of penicillin is an allergic reaction, such as skin rashes and difficulty breathing. 2. Prednisone is a glucocorticoid that has side effects as described in the stem. 3. The primary side effect of aspirin is gastritis. 4. The primary side effects of gold therapy are diarrhea, irritation of oral mucosa, and skin rashes.

A physical therapist is examining the feet of a patient with type 2 diabetes. Which of the following tests is BEST to determine the patient's risk for developing foot ulceration? 1. Pain sensation 2. Pressure threshold 3. Two-point discrimination 4. Temperature awareness

2. Pressure threshold 1. Pain sensation is not as predictive of ulceration risk as pressure threshold. 2. Pressure thresholds using nylon filaments are the most sensitive and specific assessments for risk of foot ulceration. Several studies support the use of the 10-gram (Semmes-Weinstein 5.07) nylon filament as the threshold for protective sensation. Patients unable to feel a 10-gram nylon filament are considered unable to protect their feet from injury and are at risk of ulceration. 3. Two-point discrimination is not as predictive of ulceration risk as pressure threshold. 4. Temperature awareness is not as predictive of ulceration risk as pressure threshold.

A patient is referred for recommendations regarding purchase of a wheelchair. The measurements of the patient while sitting are 16 inches (40.6 cm) across the widest point of the hips and 18 inches (45.7 cm) from the rear of the buttocks to the popliteal crease. Which of the following wheelchair dimensions would be BEST suited for this patient's needs? 1. Seat width and seat depth of 18 inches (45.7 cm) 2. Seat width of 18 inches (45.7 cm) and seat depth of 16 inches (40.6 cm) 3. Seat width and seat depth of 16 inches (40.6 cm) 4. Seat width of 16 inches (40.6 cm) and seat depth of 18 inches (45.7 cm)

2. Seat width of 18 inches (45.7 cm) and seat depth of 16 inches (40.6 cm) 2. This answer is correct because the width of this patient is 16 inches (40.6 cm), so the width of the seat should be 2 inches wider or 18 inches (45.7 cm) as stated. The depth should be 2-3 inches shorter than the distance from rear buttocks to popliteal space, which for this patient is 18 inches (45.7 cm), so seat depth of 16 inches (40.6 cm) is appropriate.

Which of the following nonsurgical interventions would be MOST appropriate for a patient who had a significant tear of the triangular fibrocartilaginous complex 2 weeks ago? 1. Carpal joint mobilization 2. Splinting for a total of 6 weeks 3. Passive radial and ulnar deviation 4. Resisted supination and pronation

2. Splinting for a total of 6 weeks 1. Carpal joint mobilization is not appropriate because it would create torque forces. 2. Conservative care includes splinting or casting. Immobilization of the wrist joint in neutral position with support against ulnar or radial deviation is suggested for 4-6 weeks. After the splint is removed, active range of motion can be initiated. Strengthening of the proximal joints can be done with the splint on, but no resistance or weight-bearing should be applied through the wrist at this point. 3. Passive range of motion is not appropriate because it creates torque forces. 4. Strengthening of the proximal joints can be done with the splint on, but no resistance or weight-bearing should be applied through the wrist at this point.

A patient has low back and leg pain, with symptoms extending to the bottom of the foot. During the physical therapy examination, the patient does not report leg pain in the first test position (photograph 1) but reports a severe increase in symptoms in the second test position (photograph 2). Which of the following conclusions is MOST likely? 1. The pain is the result of a herniated disc. 2. The patient may be displaying nonorganic symptoms. 3. The symptoms are the result of an inflamed sciatic nerve. 4. The hamstrings are in a facilitated state of contraction.

2. The patient may be displaying nonorganic symptoms. 1. If the patient had a symptomatic herniated disc, both positions would result in a similar symptom increase. 2. Because sitting knee extension and the straight-leg raise culminate in essentially identical positions, symptomatic responses to the two types of maneuvers should be similar. Taken together, the findings suggest that the patient may be displaying nonorganic symptoms. 3. If the patient had an irritated sciatic nerve, both positions would result in a similar symptom increase. 4. If the hamstrings are in a facilitated state of contraction, both positions would result in a similar symptom response.

Regular aerobic exercise lasting at least 30 minutes per session should be MOST beneficial in decreasing the symptoms associated with which of the following conditions? 1. Type 1 diabetes 2. Type 2 diabetes 3. Multiple sclerosis 4. Amyotrophic lateral sclerosis

2. Type 2 diabetes 2. Exercise is a major contributor in controlling hyperglycemia in type 2 diabetes by improving skeletal muscle glucose transport and whole-body glucose homeostasis. Regular exercise can help the body respond to insulin and is known to be effective in managing blood glucose. Exercise can lower blood glucose and possibly reduce the amount of medication needed to treat diabetes or even eliminate the need for medication

A patient reports dizziness and blurred vision when walking and turning the head to either the left or the right. The patient has no problem when the head is kept still while walking. Which of the following systems is MOST likely involved? 1. Visual 2. Vestibular 3. Somatosensory 4. Musculoskeletal

2. Vestibular

A patient has difficulty palpating the carotid pulse during exercise. The patient should be instructed in alternate methods of self-monitoring, because repeated palpation is likely to result in: 1. an increase in the heart rate. 2. a decrease in the heart rate. 3. an irregular heart rhythm. 4. an increase in blood pressure.

2. a decrease in the heart rate. 1. Heart rate is expected to decrease, not increase when the carotid sinus is overstimulated. 2. Manual pressure on the carotid sinus can cause a reflexive drop in heart rate, blood pressure, or both. 3. Heart rate, not rhythm, is expected to change due to manual pressure on the carotid sinus. 4. Blood pressure is expected to decrease, not increase, due to pressure on the carotid sinus.

A patient with chronic venous insufficiency of the lower extremities is MOST likely to exhibit: 1. normal superficial veins, no edema, ulceration, and patches of gangrene around the toes. 2. dilation of superficial veins, edema, and stasis ulceration. 3. no edema, faint dorsalis pedis pulse, and cold, hairless extremities. 4. dilation of superficial veins and edema made worse during sitting or elevation of the lower extremities.

2. dilation of superficial veins, edema, and stasis ulceration. 1. Lack of edema is not consistent with venous insufficiency. 2. With venous insufficiency, the limbs would be edematous and the superficial veins would be dilated. If the venous insufficiency is not corrected, ulceration could develop. 3. Lack of edema is not consistent with venous insufficiency. 4. Venous insufficiency should be relieved, not worsened, by leg elevation

A patient with a complete thoracic spinal cord injury is sitting in a wheelchair on a custom-made cushion. Pressure relief activities should be performed: 1. when the patient shows signs of pressure sores. 2. every 15 to 20 minutes. 3. every 1 to 2 hours. 4. if the patient does not have an appropriate cushion.

2. every 15 to 20 minutes. 1. Pressure relief should be performed before signs of skin breakdown are apparent. 2. A patient who has a thoracic-level spinal cord injury is able to perform independent pressure relief strategies and should complete pressure relief every 15 to 20 minutes. 3. Pressure relief should be performed every 15 to 20 minutes, not every 1 to 2 hours. 4. Pressure relief should be performed even with appropriate cushions. Cushions help to minimize pressure but do not eliminate it.

n the capsular pattern associated with adhesive capsulitis of the shoulder, the GREATEST percentage of loss of range of motion is in: 1. medial (internal) rotation, followed by lateral (external) rotation. 2. lateral (external) rotation, followed by abduction and medial (internal) rotation. 3. abduction, followed by lateral (external) rotation and medial (internal) rotation. 4. medial (internal) rotation, followed by abduction.

2. lateral (external) rotation, followed by abduction and medial (internal) rotation. Classic glenohumeral capsular pattern has the greatest loss of range of motion in lateral (external) rotation followed by abduction and medial (internal) rotation.

For a patient with a bilateral transfemoral amputation to maximize balance in a wheelchair, the rear wheels should be positioned more: 1. laterally. 2. posteriorly. 3. anteriorly. 4. inferiorly.

2. posteriorly. 1. Setting the wheels more laterally would create increased difficulty with propulsion of the chair and not increase stability associated with the shift of body weight due to the amputations. 2. The center of gravity of a person with bilateral transfemoral amputations is more posterior than the center of gravity of a person with lower extremities intact. Setting the back wheels more posteriorly will make the patient more stable in the chair. This adjustment prevents the wheelchair from tipping backward. 3. Setting the wheels more anteriorly would increase risk of tipping due to loss of weight associated with the amputations. 4. Setting the wheels more inferiorly may impair the patient's ability to propel the wheelchair and would not address the instability caused by the lack of body weight anteriorly.

When considering special tests for orthopedic assessment, selection of a test with adequate validity is important because the: 1. results of the test can be standardized. 2. test measures what it is supposed to measure. 3. results of the test are reproducible. 4. test can be accurately performed by someone else.

2. test measures what it is supposed to measure.

A physical therapist is evaluating the cranial nerves of a child who has a medulloblastoma. The child's right eye deviates medially. The child has impairment of which of the following cranial nerves? 1. Oculomotor (CN III) 2. Trochlear (CN IV) 3. Abducent (CN VI) 4. Vagus (CN X)

3. Abducent (CN VI) 1. The oculomotor nerve (CN III) controls the inferomedial eye muscles. 2. The trochlear nerve (CN IV) controls inferolateral eye movement. 3. The abducent nerve (CN VI) controls lateral eye movement. Damage to this nerve causes the eyeball to deviate medially due to weakness of the lateral rectus muscle. 4. The vagus nerve (CN X) does not control the eye muscles.

A patient has shoulder joint impairments. The range-of-motion examination reveals restricted lateral (external) rotation and abduction of the shoulder. Based on arthrokinematic principles, which of the following mobilization procedures should be performed for the patient FIRST? 1. Posterior glide 2. Distraction 3. Anterior glide 4. Lateral (external) rotation

3. Anterior glide 1. Based on arthrokinematic principles, posterior glides would be used when medial (internal) rotation is restricted. Posterior glides would not address the deficit in abduction. 2. For this patient, the first mobilization procedure would be distraction of the glenohumeral joint. The distraction separates the joint surfaces and is a general technique to stretch the glenohumeral capsule. Distraction may also be used in conjunction with the other mobilization techniques listed later in treatment. 3. Later mobilization techniques would most likely include anterior glide to improve lateral (external) rotation, but anterior glides would not address the abduction. 4. Mobilization into lateral (external) rotation will likely be ineffective without distraction applied to separate the joint surfaces.

A 2-year-old child has spastic quadriplegic cerebral palsy, with persistence of primitive reflexes, no selective control of movement, and frequent respiratory infections. Which of the following elements is MOST critical to include in the physical therapist's examination? 1. Modified Ashworth scale 2. Auscultation of heart sounds 3. Assessment of oral-motor control and feeding 4. Assessment of anticipatory postural adjustments

3. Assessment of oral-motor control and feeding 1. The modified Ashworth scale is used to access muscle spasticity (O'Sullivan, p. 172). Testing of the child's spasticity level should be included but is not critical. 2. Auscultation of heart sounds would not be a major component of this patient's evaluation. The individual described in the stem is at much higher risk for respiratory infections as a result of impaired oral-motor control compared to cardiac complications. 3. Oral-motor control could be a factor in respiratory infections because poor feeding is associated with weak sucking, poor coordination of the swallowing mechanism, and a hypoactive gag reflex (O'Sullivan, p. 1283). 4. Anticipatory postural adjustments are learned through movement experience, which would be absent in this child, who does not have volitional control and displays primitive reflexes

A physical therapist is preparing to evaluate a patient who had a closed reduction with cast fixation for an ankle fracture 1 day ago and is currently restricted to non-weight-bearing status. The patient reports foot pain in the injured extremity. Based on the patient's report, which of the following tests and measures should the therapist perform? 1. Gait examination 2. Transfer abilities 3. Capillary refill in toes 4. Blood pressure examination

3. Capillary refill in toes 1. Since the patient is restricted to non-weight-bearing status, transfer abilities and gait examination are less likely to yield information regarding the cause of foot pain. 2. Since the patient is restricted to non-weight-bearing status, transfer abilities and gait examination are less likely to yield information regarding the cause of foot pain. 3. With regard to capillary refill in toes, complications of cast fixation may include swelling, and if severe, compartment syndrome. Improperly fitted casts and/or prolonged positioning of the limb in a dependent position can result in painful swelling that creates occlusion to distal blood flow. Testing capillary refill is an easy way to examine distal extremity perfusion. 4. Blood pressure changes could cause changes in perfusion to distal extremities, but it would be very difficult to measure lower extremity blood pressure with the cast on.

A physical therapist is working with a child who recently learned to sit independently. Which of the following gross motor skills should be the focus of intervention NEXT? 1. Cruising 2. Walking 3. Creeping on hands and knees 4. Rolling from prone to supine position

3. Creeping on hands and knees 1. Creeping would typically occur before cruising. 2. Creeping and cruising would typically occur before walking. 3. Creeping on hands and knees is expected to come after independent sitting. 4. Rolling would have already been achieved because children learn to roll prior to sitting.

A patient reports weakness and tingling in the lower extremities over the past 2 weeks. The physical therapist suspects that the patient may have Guillain-Barré syndrome. Which of the following examination findings would MOST likely occur with this diagnosis? 1. Hypertonicity in the affected muscles 2. Presence of clonus with rapid passive foot dorsiflexion 3. Diminished tendon reflexes 4. Ataxic gait pattern

3. Diminished tendon reflexes 1. Hypertonicity is expected with upper motor neuron disease. Guillain-Barré syndrome is a lower motor neuron disease and is characterized by flaccidity. 2. Clonus is expected with upper motor neuron disease. Guillain-Barré syndrome is a lower motor neuron and is characterized by hyporeflexia. 3. Guillain-Barré syndrome is a lower motor neuron disorder. Diminished reflexes are expected with lower motor neuron disease. 4. Ataxia would be much more common with cerebellar disorders.

Which of the following mobilization techniques for the radiocarpal joint would be MOST appropriate for increasing limited wrist flexion range of motion? 1. Radial glide 2. Ulnar glide 3. Dorsal glide 4. Volar glide

3. Dorsal glide 1. Radial glides improve ulnar deviation (p. 820). 2. Ulnar glides improve radial deviation (p. 820). 3. Dorsal glides improve wrist flexion based on the convex-concave rule (pp. 819-820). 4. Volar glides improve wrist extension (p. 820).

A patient with which of the following diagnoses would MOST likely benefit from pursed-lip breathing during exercise? 1. Peripheral vascular disease 2. Heart failure 3. Emphysema 4. Sarcoidosis

3. Emphysema 1. Peripheral vascular disease is a vascular problem, not a pulmonary problem. 2. Heart failure may lead to pulmonary problems, but not to obstructive problems that could be addressed by pursed-lip breathing. 3. Emphysema is an obstructive lung disease for which pursed-lip breathing may be beneficial. 4. Sarcoidosis is a restrictive lung disease for which pursed-lip breathing is not beneficial.

Which of the following interventions is MOST appropriate for a patient who has juvenile rheumatoid arthritis and is experiencing painful swelling of both knees? 1. Resistive exercises 2. Stretching to prevent contractures 3. Gentle, active exercises 4. Walking program

3. Gentle, active exercises 1. Painful swelling is indicative of an acute exacerbation. Resistive exercises are contraindicated during an acute exacerbation to protect the joint (p. 333). 2. Painful swelling is indicative of an acute exacerbation. Stretches are contraindicated during an acute exacerbation to protect the joint (pp. 333-334). 3. Gentle, active exercises should be encouraged during an acute exacerbation of juvenile rheumatoid arthritis (p. 334). 4. Participation in a walking program would not be most appropriate during an acute exacerbation of juvenile rheumatoid arthritis affecting the knees (p. 334).

A physical therapist is evaluating a patient who has a chest tube. The therapist accidentally knocks over the collection reservoir. After returning it to an upright position, which of the following locations is MOST appropriate for the collection reservoir? 1. Hang it on an intravenous pole. 2. Place it on the patient's bedside table. 3. Hang it from the side of the bed. 4. Place it on the bed near the insertion site.

3. Hang it from the side of the bed. 3. Hanging the collection reservoir from the side of the bed or securing it on the floor are recommended. These positions keep the drainage system below the level of the heart and prevent tipping of the reservoir.

A patient sustained a gunshot wound to the spine in the area of L1. The patient has weakness of the left lower extremity and inability to move the knee, ankle, or foot. The patient's patellar tendon and Achilles tendon reflexes are increased on the left side. There is also loss of proprioception in the patient's left ankle and knee, a positive Babinski sign on the left side, and diminished sensation to pinprick and temperature changes in the right thigh, leg, and foot. Results of all of the patient's cranial nerve tests are normal. These findings are consistent with which of the following injuries? 1. Complete severance of the spinal cord 2. Injury to the left anterior horn of the spinal cord 3. Injury to the left side of the spinal cord 4. Injury to the central area of the spinal cord

3. Injury to the left side of the spinal cord 1. Complete severance of the spinal cord causes motor and sensory loss on both sides of the body (Umphred, p. 461). 2. A lesion of the anterior horn cells causes a lower motor neuron problem and hyporeflexia (Martin, p. 21). 3. The signs and symptoms presented in the question point to a hemisection of the spinal cord on the left side, which gives rise to ipsilateral motor and proprioception loss as well as contralateral loss of pain and temperature. Also present are hyperreflexia and a positive Babinski sign on the left side. (Umphred, p. 462) 4. A lesion of the central spinal cord usually spares the motor tracts (Umphred, p. 462).

The physical therapy plan for a patient who underwent a medial meniscectomy includes exercising the quadriceps femoris against accommodating resistance. Which of the following exercises should be recommended? 1. Knee bends through half of the range 2. Knee extension with an ankle weight 3. Knee extension on an isokinetic exercise device 4. Straight-leg raising with a sling suspensio

3. Knee extension on an isokinetic exercise device 3. Isokinetic exercise devices utilize accommodating resistance throughout the range of motion (pp. 184-185). ^ this is the only option that allows accommodating resistance

A physical therapist is working with a patient who has a complete T6 spinal cord injury. The therapist has the patient perform seated push-ups on a mat by having the patient push down on the mat with both upper extremities while attempting to lift the buttocks off the mat. This activity is aimed at strengthening which muscle group? 1. Quadratus lumborum 2. Internal obliques 3. Latissimus dorsi 4. External obliques

3. Latissimus dorsi 1. The quadratus lumborum is innervated below the level of the lesion (Hislop, p. 49). 2. The internal obliques are innervated below the level of the lesion (Hislop, p. 49). 3. Sitting push-ups are often used as preparation for gait training and transfers. The push-ups are done to strengthen the muscles that "hike the pelvis" or lift the buttocks from the mat in a seated position. In a patient who has a complete spinal cord injury at the T6 level, the only muscle group that is still capable of lifting the pelvis is the latissimus dorsi, which is innervated by the cervical roots C6-C8. The therapist is attempting to strengthen these muscles because they are capable of lifting the pelvis in the absence of the erector spinae and abdominal musculature, which are innervated below the level of the lesion (Hislop, pp. 105, 458). 4. The external obliques are innervated below the level of the lesion (Hislop, p. 49).

Which of the following positions is MOST effective to actively stretch the lumbrical muscles? 1. Metacarpophalangeal and interphalangeal flexion 2. Metacarpophalangeal and interphalangeal extension 3. Metacarpophalangeal extension and interphalangeal flexion 4. Metacarpophalangeal flexion and interphalangeal extension

3. Metacarpophalangeal extension and interphalangeal flexion The lumbricals contract to a shortened position of metacarpophalangeal flexion and interphalangeal extension. Thus, metacarpophalangeal extension and interphalangeal flexion would best stretch the muscles.

A 65-year-old patient reports left hip morning stiffness and hip pain that increases with medial (internal) rotation. The patient demonstrates restriction in hip motion for both medial (internal) rotation and flexion. These signs and symptoms are MOST likely caused by which of the following conditions? 1. Neoplasm 2. Fibromyalgia 3. Osteoarthritis 4. Rheumatoid arthritis

3. Osteoarthritis Hip osteoarthritis is characterized by hip pain that increases with 15° of hip medial (internal) rotation, and morning stiffness that lasts approximately 1 hour. It typically occurs in patients older than age 50 years. Hip medial (internal) rotation and hip flexion are commonly limited as the osteoarthritis progresses. Literature indicates that this presentation has an 86% sensitivity and 75% specificity for hip osteoarthritis.

A patient with Guillain-Barré syndrome who is experiencing progressive paralysis is admitted to an intensive care unit. Which of the following interventions is MOST appropriate for a physical therapist to perform as a member of the interdisciplinary team managing the patient's care? 1. Alter the patient's mechanical ventilation settings. 2. Develop the patient's medication schedule. 3. Perform airway clearance techniques. 4. Discuss the patient's medical prognosis with the family.

3. Perform airway clearance techniques. 1. Altering the mechanical ventilator settings would be outside the scope of practice for a physical therapist. 2. Developing a medication schedule would be outside the scope of practice for a physical therapist. 3. The key term in this question is "interdisciplinary." Of all the options provided, only airway clearance is within the scope of practice for physical therapists. Patients who have Guillain-Barré syndrome may experience respiratory muscle fatigue or paralysis and are susceptible to pulmonary infection. Pulmonary hygiene is a critical role in their care. 4. Discussing the medical prognosis would be outside the scope of practice for a physical therapist.

A physical therapist is evaluating a patient who has a vascular lesion in the brainstem affecting the oculomotor nerve (CN III). During the cranial nerve examination, which of the following signs would be the MOST significant? 1. Inability to close the eyelid 2. Medial strabismus 3. Ptosis of the eyelid 4. Difficulty looking inferomedially

3. Ptosis of the eyelid 1. Inability to fully close the eye would be seen with a lesion of the facial nerve (CN VII) (Bell palsy). 2. Medial strabismus would be caused by damage to the abducent nerve (CN VI), innervating the lateral rectus, causing a medial strabismus. 3. The oculomotor nerve (CN III) innervates the levator palpebrae superioris muscle which elevates the upper eyelid, and the pupillary constrictor muscle. Therefore, a lesion of the oculomotor nerve would make it difficult, if not impossible, for the patient to fully raise the lid (open the eye) and would cause a condition termed ptosis. 4. Difficulty looking inferomedially is due to a trochlear nerve (CN IV) problem.

A physical therapist observes a patient from behind during bilateral shoulder abduction and notes that the patient's right scapula is more abducted than the left scapula at the end range of movement. Which of the following conditions is the MOST likely cause of the altered scapula position on the right? 1. Tightness of the rhomboid major and minor 2. Weakness of the serratus anterior 3. Restricted motion of the glenohumeral joint 4. Weakness of the upper trapezius

3. Restricted motion of the glenohumeral joint 1. Tightness of the rhomboid major and minor would promote downward rotation of the scapula. 2. Weakness of the serratus anterior would limit the upward rotation of the scapula. 3. The most likely reason for the increase in scapular motion is restriction of the glenohumeral joint. To fully abduct the shoulder, the scapula and glenohumeral joint both have to contribute to produce the motion. If the glenohumeral joint is restricted, the scapula has to increase its motion to accomplish the task. 4. Weakness of the upper trapezius would result in a scapular lag in upward rotation.

A patient involved in a motor vehicle accident 1 week ago has multiple fractures of the lower extremities and has bilateral non-weight-bearing status. The patient lives with an elderly parent and is scheduled for discharge soon. Which of the following transfers is MOST appropriate in this situation? 1. Standing pivot 2. One-person lift 3. Sliding board 4. Two-person lift

3. Sliding board The sliding-board transfer is the best choice of the options because the caregiver is elderly and the patient is non-weight-bearing bilaterally.

A runner is referred to physical therapy with medial ankle pain. Examination shows posterior medial malleolar tenderness and a supinated hindfoot at heel strike (initial contact). Which of the following muscles is MOST likely responsible for these impairments? 1. Tibialis anterior 2. Soleus 3. Tibialis posterior 4. Fibularis (peroneus) longus

3. Tibialis posterior 3. Supination involves combined movements of inversion, adduction, and plantar flexion. The tibialis posterior crosses the medial side of the ankle joint posterior to the malleolus. It is an invertor and plantar flexor of the foot. (p. 355)

A patient has an abscess on the left thoracic region just distal to the left scapula that has been surgically incised to drain the abscess. The patient has a tubular defect that travels 2.4 in (6.1 cm) transversely under the skin toward the right scapula. With the patient in left sidelying position, which of the following statements MOST accurately represents the correct documentation of the defect? 1. Tunneling at 12 o'clock 2. Undermining at 12 o'clock 3. Tunneling at 3 o'clock 4. Undermining at 3 o'clock

3. Tunneling at 3 o'clock Undermining is tissue destruction that occurs under intact skin around the wound perimeter. The undermining options are incorrect, because the wound described in the item stem is a channel, not a separation around the wound's perimeter. A tunnel is a channel that extends from any part of the wound through subcutaneous tissue or muscle. Undermining and tunneling are documented by measuring depth and noting the location using the face of the clock as a guide. Using the clock method, the patient's head is oriented at 12 o'clock and the feet toward 6 o'clock. Tunneling at 3 o'clock is correct, because the channel extends toward 3 o'clock.

The MOST appropriate short-term goal for a patient with lateral epicondylalgia would be pain-free isometric contraction of which muscle group? 1. Forearm supinators 2. Wrist flexors 3. Wrist extensors 4. Forearm pronators

3. Wrist extensors 3. The primary structures involved in lateral epicondylalgia are the extensor carpi radialis brevis and extensor communis; therefore, pain-free isometric contractions of the wrist extensors would be an appropriate goal. -all other options are not involved with lateral epicondylgia

When planning a physical therapy program for a patient who has early stage Alzheimer disease, it is MOST important that the daily activities: 1. promote involvement and interest through maximum stimulation. 2. are changed each day to meet the need for variety. 3. are highly structured to reduce anxiety and confusion. 4. provide frequent opportunities for decision making.

3. are highly structured to reduce anxiety and confusion. 1. Patients who have Alzheimer dementia are likely to respond poorly to highly stimulating environments (p. 148). 2. Patients who have Alzheimer dementia are likely to respond poorly to unnecessary change in daily activities (p. 148). 3. Patients who have Alzheimer dementia benefit from a structured, soothing environment that is designed to ensure safety and optimal function (p. 149). 4. Patients who have Alzheimer dementia are likely to respond poorly to situations requiring greater cognitive skills than they possess, such as frequent decision making

Which of the following ankle-brachial index values would indicate an absolute CONTRAINDICATION to compression therapy? 1. 1.3 2. 1.0 3. 0.8 4. 0.5

4. 0.5 4. Interpretation of the ankle-brachial index is critical in selecting appropriate intervention. Application of compression to a limb that demonstrates severe arterial occlusive disease may lead to limb ischemia and necrosis. An ankle-brachial index of 0.5 to 0.8 indicates moderate arterial occlusive disease. Compression therapy is contraindicated when the ankle-brachial index is less than 0.8.

A patient who has a subacute stroke is being admitted to a rehabilitation hospital. Which of the following conditions is associated with the BEST prognosis for improved mobility? 1. Admission to rehabilitation at 1 month post stroke 2. Hemispheric lesion as compared to a lacunar lesion 3. Impairments in production and comprehension of speech 4. Ability to sit independently upon admission to rehabilitation

4. Ability to sit independently upon admission to rehabilitation 1. Timing of rehabilitation services within the first 20 days of onset of stroke has been shown to improve outcomes as compared to longer intervals (i.e., 1 month) (O'Sullivan, p. 666). 2. Patients who have lacunar strokes demonstrate improved motor recovery as compared to patients who have hemispheric lesions (O'Sullivan, p. 658). 3. Global aphasia often impedes successful outcomes in rehabilitation (O'Sullivan, p. 656). 4. Sitting balance has been shown to be a good prognostic indicator of outcome for persons who have had a stroke

Which of the following examination findings is indicative of developing nerve root signs? 1. Upgoing Babinski sign 2. Absent Hoffman sign 3. Cogwheel weakness with strength testing 4. Fading deep tendon reflex with repetitive tapping

4. Fading deep tendon reflex with repetitive tapping 1. The Babinski sign is suggestive for central nervous system dysfunction. 2. The Hoffman sign is an indicator of cervical myelopathy or upper motor neuron disease. A negative or absent sign suggests these conditions do not exist. 3. Cogwheel rigidity is a sign of central nervous system dysfunction. 4. A fading deep tendon reflex with repetitive tapping indicates developing nerve root signs

Small-amplitude oscillations performed at the limit of the available motion and into tissue resistance would be MOST appropriate to address which of the following end-feels? 1. Bony 2. Empty 3. Spasm 4. Capsular

4. Capsular 1. Manual techniques are not appropriate for bony end-feels. 2. Manual techniques are not appropriate for empty end-feels. 3. Manual techniques are not appropriate for spasm end-feels. 4. Small-amplitude oscillations performed at the limit of the available motion and into tissue resistance describes Grade IV joint mobilization. Grade IV mobilizations are appropriate to address restrictions associated with capsular end-feels.

What precautions should a physical therapist observe when working with a patient infected with methicillin-resistant Staphylococcus aureus? 1. Airborne 2. Sterile 3. Droplet 4. Contact

4. Contact Methicillin-resistant Staphylococcus aureus is spread by contact.

The physical therapy intervention for a patient with dark skin pigmentation includes superficial heat. After 5 minutes of this intervention, the physical therapist removes the heat and observes that the skin over the region being treated is blotchy with both darker and lighter areas. What is the MOST appropriate action for the therapist to take? 1. Discontinue superficial heat and initiate ultrasound. 2. Continue with the use of superficial heat, because this is a normal response. 3. Discontinue superficial heat and initiate ice application. 4. Continue with the use of superficial heat, but add more towel layers.

4. Continue with the use of superficial heat, but add more towel layers. 4. Uneven, blotchy, red, and white skin in a light-skinned patient or darker and lighter areas in a darker-skinned patient could be a sign of overheating. Five minutes after initiating heating, the physical therapist should remove the pack and check the patient's skin for any blotching-mottled erythema. If it is found, the heat is too intense, and adding more towel layers would be the most appropriate action.

A physical therapist is evaluating a patient with low back pain and associated symptoms. Which of the following findings should cause the therapist to refer the patient back to the physician? 1. Atrophy of the gluteal mass 2. Pain radiating down the back of the thigh into the posterior lower leg 3. Straight leg raise of 45° and hip flexion of 70° 4. Decreased perineal reflexes

4. Decreased perineal reflexes 1. Atrophy of the gluteal mass is associated with S1-S3 nerve root involvement. It does not warrant sending the patient back to the physician. 2. Pain down the back of the thigh and calf can be associated with L4 or L5 involvement. It does not warrant sending the patient back to the physician. 3. If the straight leg raise was associated with no increase in hip flexion once the knee was bent, referral to the physician would be necessary, as this is a positive sign of the buttock 4. Decreased perineal reflexes are associated with cauda equina syndrome, which requires further evaluation by a physician.

A patient suspected of having hypoglycemia is MOST likely to show which of the following signs? 1. Fruity smelling breath 2. Thirst, nausea, and vomiting 3. Dry, crusty mucous membranes 4. Difficulty speaking and concentrating

4. Difficulty speaking and concentrating 1. Fruity smelling breath is a sign of hyperglycemia and not a sign of hypoglycemia. 2. Thirst, nausea, and vomiting are all signs of hyperglycemia and not signs of hypoglycemia. 3. Dry, crusty mucous membranes are a sign of hyperglycemia and not a sign of hypoglycemia. 4. A common mental state manifestation of hypoglycemia is difficulty speaking and concentrating.

Which of the following locations corresponds to the sensory function of the nerve root exiting below the fifth lumbar vertebra? 1. Plantar aspect of the heel 2. Central anterior distal thigh 3. Medial aspect of the knee joint 4. Dorsal web space between the first and second toes

4. Dorsal web space between the first and second toes 1. The plantar aspect of the heel is supplied by S1. 2. The central anterior distal thigh is supplied by L4. 3. The medial aspect of the knee joint is supplied by L3. 4. The L5 nerve root exits below the fifth lumbar vertebra and supplies sensory information from the dorsal aspect of the web space between the first and second toes.

A patient with an L4-L5 posterolateral herniated nucleus pulposus is MOST likely to have sensory deficits in which of the following locations? 1. Medial knee 2. Over the popliteal fossa 3. Plantar aspect of the fifth toe 4. Dorsum of the great toe

4. Dorsum of the great toe 1. The medial knee is part of the L3 dermatome (p. 141). 2. The popliteal fossa is part of the S2 dermatome (p. 141). 3. The plantar foot is part of the S1 or S2 dermatome (p. 141). 4. An intervertebral disc protrusion compresses the nerve root numbered one spinal level inferior to the herniated disc. The nerve root numbered one spinal level superior to the herniated disc exits via the superior bony half of the intervertebral foramen and thereby avoids compression from the herniated disc below. Therefore, a posterolateral disc bulge at L4-L5 will most likely affect the L5 nerve root and the dermatome for the L5 nerve root. The L5 dermatome includes the dorsal aspect of the great toe. (pp. 141, 1494)

What is the close-packed position of the radiocarpal joint of the wrist? 1. Flexion with ulnar deviation 2. Flexion with radial deviation 3. Extension with ulnar deviation 4. Extension with radial deviation

4. Extension with radial deviation The close-packed position for the radiocarpal joint is extension with radial deviation or full extension.

The test shown in the photograph is used to identify which of the following conditions? 1. Hip osteoarthritis 2. Iliopectineal bursitis 3. Acetabular labral tear 4. Femoral shaft stress fracture

4. Femoral shaft stress fracture 1. The fulcrum test is not used to assess for hip osteoarthritis. The FABER (flexion, abduction, and lateral (external) rotation test), or Patrick test, would be more likely to elicit symptoms related to this condition. 2. The fulcrum test is not used to identify iliopectineal bursitis. Resisted hip flexion and passive hip extension would be more likely to elicit symptoms related to this condition. 3. The fulcrum test is not used to identify acetabular labral tears. The scour test would be used to test for this condition. 4. The test shown in the photograph is the fulcrum test, which is used to detect femoral shaft stress fractures. A positive test result is indicated by the patient's report of pain or apprehension when pressure is applied.

Which of the following descriptions is MOST appropriate for a healed wound with thick fibrous tissue that remains within the original wound border? 1. Epibole 2. Keloid scarring 3. Lichenification 4. Hypertrophic scarring

4. Hypertrophic scarring 1. Epibole is used to describe a condition in which the wound edges are rolled under and the wound bed remains open. 2. Keloid scarring is a condition in which excessive scar tissue grows outside of the original margins of the wound. 3. Lichenification is a term used to describe skin that becomes hard and leathery. Its texture is similar in appearance to lichens. 4. Hypertrophic scarring would describe a healed wound with thick fibrous tissue that remains within the original wound border

A patient recovering from a cervical spine injury is performing stabilization exercises that focus on the lower cervical and upper thoracic extensor muscles. Which of the following exercises would provide the GREATEST cervical spine protection? 1. In prone position, elevate the upper extremities in full flexion. 2. In standing position, reach forward, outward, and upward in functional patterns. 3. In standing position with no support, perform pushing, pulling, and lifting activities. 4. In prone position with the upper extremities abducted and laterally (externally) rotated, horizontally abduct the shoulders and adduct the scapulae.

4. In prone position with the upper extremities abducted and laterally (externally) rotated, horizontally abduct the shoulders and adduct the scapulae. 1. Elevating the shoulders in prone position provides less protection to the cervical spine than the upper extremities abducted and laterally (externally) rotated, the shoulders horizontally abducted, and the scapulae adducted. 2. Prone position provides greater cervical spine support than standing position. 3. Prone position provides greater cervical spine support than standing position. 4. Prone position provides greater cervical spine support than standing position. Maximal protection is provided by having the upper extremities abducted and laterally (externally) rotated, the shoulders horizontally abducted, and the scapulae adducted, whereas elevating the shoulders in prone position provides moderate to minimum protection.

Which of the following postural drainage positions is MOST appropriate for a patient who has aspiration pneumonia in the right middle lobe and who had a craniotomy 2 days ago? 1. Right sidelying with one-quarter turn toward supine and the bed flat 2. Left sidelying with one-quarter turn toward supine with head of bed down approximately 20° 3. Right sidelying with one-quarter turn toward supine with head of bed down approximately 20° 4. Left sidelying with one-quarter turn toward supine and the bed flat

4. Left sidelying with one-quarter turn toward supine and the bed flat The traditional position to drain the right middle lobe is left sidelying with one-quarter turn toward supine position, with the head of the bed down approximately 20°. This position does not take into account the intracranial pressure issues. Because the patient recently underwent a craniotomy, intracranial pressure should be monitored and kept stable. Therefore, this traditional position should be modified to sidelying without declining the head of the bed.

Which of the following modalities is MOST appropriate for decreasing pain and increasing tissue extensibility prior to active hand exercises in a patient who has rheumatoid arthritis? 1. Cold gel pack 2. Direct contact ultrasound 3. Neuromuscular electrical stimulation 4. Paraffin wax bath

4. Paraffin wax bath 4. Paraffin wax baths are used to decrease pain and increase tissue extensibility in patients who have rheumatoid arthritis. The increase in collagen extensibility associated with heating may decrease pain perception and increase tolerance for participation in active exercise. Paraffin allows for even distribution of heat to fingers

Which of the following endocrine glands regulates sexual development? 1. Parathyroid 2. Thyroid 3. Adrenal 4. Pituitary

4. Pituitary The parathyroid gland is responsible for calcium and phosphorus metabolism and bone calcification. It does not play a role in sexual development. 2. Secretion of hormones via the thyroid gland is regulated by the pituitary gland. The thyroid secretes hormones that control metabolism and protein synthesis. It also influences calcium and phosphorus metabolism. The thyroid gland does not play a role in sexual development. 3. Hormone secretion by the adrenal glands is regulated by the pituitary gland. The adrenal gland does secrete hormones responsible for secondary sexual characteristics. However, it is primarily involved in fluid/electrolyte balance and metabolism. 4. The pituitary gland serves as a master gland and regulates the secretion of many hormones. The anterior pituitary glands regulate sexual development via release of gonadotropins. Gonadotropins regulate secretion of male and female hormones.

The work modification (standing) shown in the photograph is MOST appropriate for a patient with which of the following conditions? 1. Central lumbar stenosis 2. Deep vein thrombosis 3. Thoracolumbar scoliosis 4. Posterolateral lumbar disc bulge

4. Posterolateral lumbar disc bulge 1. A patient who has stenosis will do better in sitting position, not standing (Dutton, p. 1499). 2. Preference for lumbar extension is not associated with deep vein thrombosis (Goodman, p. 261). 3. Preference for lumbar extension is not associated with scoliosis (Dutton, p. 1576). 4. The photograph shows a standing desk/work station. Sitting increases intradiscal pressure, so standing is often preferred to sitting for patients who have a posterolateral lumbar disc bulge. (Dutton, p. 1484)

A patient who has atrophy of the hypothenar eminence of the hand will most likely have the GREATEST difficulty performing which of the following tasks? 1. Holding a cup of water and transferring the contents into another cup 2. Writing two short sentences on a piece of paper using a fine-tipped pen 3. Making a circle with the fingers by touching the tips of the thumb (1st digit) and the index finger (2nd digit) 4. Pulling a piece of paper between the pads of the thumb (1st digit) and the index finger (2nd digit)

4. Pulling a piece of paper between the pads of the thumb (1st digit) and the index finger (2nd digit) 4. The ulnar nerve innervates the muscles in the hypothenar aspect of the palm of the hand, namely the flexor digiti minimi, abductor digiti minimi, and opponens digiti minimi. In addition, the ulnar nerve innervates other intrinsic muscles, including the interossei, the two medial lumbrical muscles, and the adductor pollicis. These muscles function to keep the metacarpals tightly together when performing gripping tasks, making the ulnar nerve responsible for power of the hand. However, by virtue of its innervation of the adductor pollicis, it is also responsible for adduction of the thumb (1st digit), which is required when pulling an object with the thumb and index finger (1st and 2nd digits), such as the piece of paper. Without the adductor pollicis, the thumb (1st digit) flexors will attempt to substitute, and this will result in inadvertent flexion of the thumb (1st digit), losing the grip between the thumb and index finger (1st and 2nd digits). In formal testing, this is referred to as Froment sign.

When performing a supine to long-sit test, a patient's right lower extremity is found to be shorter in supine position and longer in sitting position. Which of the following rotations is occurring at the pelvis? 1. Left anterior rotation of the ilium 2. Right anterior rotation of the ilium 3. Left posterior rotation of the ilium 4. Right posterior rotation of the ilium

4. Right posterior rotation of the ilium A posterior rotation will occur on the involved leg (right) during sacroiliac joint involvement. As a result, the leg at the side of the posterior rotation will be shorter in supine position and longer in sitting position.

A patient is referred to a physical therapist for treatment of left shoulder pain. During examination of the patient's left acromion, the therapist notes an area of skin discoloration that is 1.3 cm in diameter and lopsided in shape with irregular, poorly defined edges. The center is deep red with some shades of brown. The patient reports first noticing the discoloration about 2 months ago. The therapist should recognize these signs as characteristic of: 1. basal cell carcinoma. 2. squamous cell carcinoma. 3. a nevi. 4. a malignant melanoma.

4. a malignant melanoma. 1. Basal cell carcinomas usually occur on hair-bearing areas exposed to the sun, such as the face, neck, head, ears, or hands. They grow slowly to 1-2 cm after years of growth. This does not fit the patient's presentation. 2. Squamous cell carcinoma typically appears on the ear, face, lips, mouth, and dorsa of the hands. They are small, red, hard nodules. 3. Nevi can have irregular borders and colors, but are usually smaller than 6 mm in diameter and have uniform pigmentation. 4. Malignant melanoma can appear anywhere in the body. It is usually asymmetric and has irregular borders, and its colors can be black, brown, red, or white. It is larger than a pencil eraser, and it evolves in shape. Characteristics noted in the stem fit this description.

A physical therapist is conducting a reflex text as shown in the photographs. The arrow indicates the path of the applied stimulus. The results of the test are MOST likely to indicate: 1. a peripheral nerve injury. 2. a lesion of the anterior horn cells. 3. a normal response. 4. an injury to the spinal cord.

4. an injury to the spinal cord. 1. The presence of a positive Babinski sign indicates an upper motor neuron lesion such as a spinal cord injury. Peripheral nerve injuries are lower motor neuron lesions. 2. The presence of a positive Babinski sign indicates an upper motor neuron lesion such as a spinal cord injury. Lesions of the anterior horn cells are lower motor neuron lesions. 3. The presence of a positive Babinski response is indicative of a upper motor neuron lesion, which is not normal. If no upper motor neuron lesion is present, the toes will flex. 4. The therapist is testing for a positive Babinski sign. When the Babinski sign is positive, the toes flare at the end of the test. The presence of a positive Babinski sign indicates an upper motor neuron lesion such as a spinal cord injury.

On evaluation, a patient with dorsal column syndrome would be expected to exhibit: 1. left extremity weakness and hyperesthesia. 2. bilateral extremity weakness and hypoesthesia. 3. balance instability and decreased intersegmental trunk rotation. 4. decreased vibratory sensation and decreased proprioception.

4. decreased vibratory sensation and decreased proprioception. 1. Dorsal column syndrome does not have a motor component that would decrease extremity strength. 2. Dorsal column syndrome does not have a motor component that would decrease extremity strength. 3. Dorsal column syndrome does not have a motor component that would decrease intersegmental trunk rotation. 4. Dorsal column syndrome causes decreased perception of vibration and proprioception.

A physical therapist is measuring a patient's ankle dorsiflexion range of motion. The measurement shown in photograph A is greater than the measurement shown in photograph B. The MOST likely cause of the difference in measurement is: 1. talocrural joint capsule restriction. 2. weakness of the ankle dorsiflexors. 3. tightness of the hamstrings. 4. tightness of the gastrocnemius.

4. tightness of the gastrocnemius. 1. Capsular restriction would affect both measurements. 2. Dorsiflexor weakness would affect both measurements. 3. Hamstring tightness would affect knee range of motion, not ankle range of motion. 4. In photograph A, there is more dorsiflexion present with the knee bent. In this position, the gastrocnemius is on slack across the knee joint, which allows greater range of dorsiflexion. In photograph B, the gastrocnemius is stretched over the knee joint, so tightness in the gastrocnemius would restrict ankle dorsiflexion.

Which of the following assignments is MOST appropriate for a physical therapist to delegate to a volunteer? 1. Restocking treatment booths with linens, ultrasound gel, and massage lotion 2. Attending a patient who is exercising on a recumbent bike while the therapist takes a phone call 3. Transporting a patient who reports dizziness back to the patient's room 4. Assisting a patient in transferring from the mat table to a wheelchair

1. Restocking treatment booths with linens, ultrasound gel, and massage lotion 1. The restocking of treatment booths with supplies is the only option that does not involve direct patient contact or care and thus is the MOST appropriate activity to delegate to a volunteer. 2. Although volunteers may at times be involved with patient care activities (i.e., transporting patients), a patient who is exercising is potentially at risk and would require supervision by the physical therapist. 3. A volunteer should not be asked to transport a patient who is potentially unstable. 4. Although volunteers may at times be involved with patient care activities (i.e., transporting patients), a volunteer should not direct patient transfers.

During a five-position grip strength test, the HIGHEST grip strength measurement is MOST likely to result from use of which handle position of a hand-held dynamometer? 1. 1 2. 2 3. 4 4. 5

2. 2 2. The widest grips (4 and 5) use mostly the flexor digitorum profundus muscle; the smallest grip (1) prevents much use of the flexor digitorum profundus and superficialis because they are maximally shortened. In healthy and motivated patients, maximum grip strength occurs at the second or third grip width.

A physical therapist is educating a patient on the use of a moist hot pack for home treatment. For the patient to prevent burns and still receive the benefits of superficial heat, which of the following heat application time frames is MOST appropriate? 1. 5-10 minutes 2. 20-30 minutes 3. 45-60 minutes 4. 61-90 minutes Your Answer: 1 Correct Answer: 2 You have incorrectly answered the question.

2. 20-30 minutes 1. Five to 10 minutes is an insufficient amount of time for therapeutic heating effects. 2. The ideal amount of time for therapeutic heating effects with minimal risk of burns is 20-30 minutes. 3. Forty-five to 60 minutes is too long a period of time, and could increase the risk of burn. 4. Sixty-one to 90 minutes is too long and presents a significantly increased risk of burn.

A physical therapist is measuring a patient's passive knee range of motion. The measurements obtained are shown in photographs A and B. The MOST likely cause of the difference in knee range of motion is: 1. restriction in the knee joint capsule. 2. tightness of the rectus femoris. 3. weakness of the hamstrings. 4. tightness of the vastus medialis.

2. tightness of the rectus femoris. 2. In photograph A, there is more knee flexion present with the hip flexed. In this position, the rectus femoris is on slack across the hip joint, allowing greater range of knee flexion. In photograph B, the rectus femoris is stretched over both the knee joint and the hip joint, so tightness in the rectus femoris would restrict knee flexion. Photograph B also shows hip joint flexion

A patient with a positive Finkelstein test reports pain over the radial aspect of the wrist that is aggravated by frequent lifting of heavy cooking pots. Which tendons are PRIMARILY involved? 1. Extensor carpi radialis longus and adductor pollicis 2. Flexor carpi radialis and flexor pollicis longus 3. Abductor pollicis longus and extensor pollicis brevis 4. Extensor pollicis longus and extensor pollicis brevis

3. Abductor pollicis longus and extensor pollicis brevis The tendons tested by the Finkelstein test are the tendons of the abductor pollicis longus and the extensor pollicis brevis.

A physical therapist has been working with a patient who has a spinal cord injury. To document that the patient has been educated about skin care, the therapist should record that the patient: 1. is unable to tolerate more than 1 hour in a wheelchair. 2. has good strength of the scapular depressors and elbow extensors. 3. is able to state three causes of skin breakdown. 4. is able to perform 10 push-ups in the wheelchair.

3. is able to state three causes of skin breakdown. According to the National Safety Foundation, patient's education is most effective when the patient can respond to each of these questions: 1) What is my main problem? 2) What do I need to do? 3) Why is it important for me to do this? A record that the patient is able to state causes of skin breakdown answers the first of three questions.

A patient with cervical radiculopathy reports numbness of the right little finger (5th digit). A physical therapist will MOST likely find a diminished tendon reflex in the: 1. biceps brachii. 2. deltoid. 3. triceps brachii. 4. brachioradialis.

3. triceps brachii. 1. The dermatome providing sensation to the little finger is innervated by the C8 nerve root. The biceps brachii is innervated by the C5, C6, and sometimes C7 nerve roots. 2. The dermatome providing sensation to the little finger is innervated by the C8 nerve root. The deltoid is innervated by the C5, C6, and sometimes C7 nerve roots. 3. The dermatome providing sensation to the little finger is innervated by the C8 nerve root. The triceps brachii is the only one of the muscles listed that is also innervated by the C8 nerve root. 4. The dermatome providing sensation to the little finger is innervated by the C8 nerve root. The brachioradialis is innervated by the C5, C6, and sometimes C7 nerve roots.

A patient who is transported to the physical therapy department in a wheelchair reports severe, bilateral lower extremity pain. A purple discoloration of both feet is observed. The pain is relieved when the patient's feet are raised just above the horizontal plane. These signs are MOST indicative of: 1. arterial insufficiency. 2. intermittent claudication. 3. venous insufficiency. 4. a psychosomatic episode.

3. venous insufficiency. 1. With arterial insufficiency, elevation increases ischemia and, therefore, pain. 2. Intermittent claudication is a phenomenon associated with metabolic demands exceeding the capability of the vascular system to supply adequate blood flow and manifests as pain during lower extremity exercise. 3. With venous insufficiency, placing the limb in a dependent position increases swelling and, therefore, possibly, pain. 4. An objective sign, such as purple discoloration, rules out a psychosomatic episode.

Following a cerebrovascular accident, a patient is evaluated for cognitive and perceptual dysfunctions. The patient is asked to stack several wooden blocks. After picking up a block, the patient is unable to determine how the block should be used. This dysfunction is MOST likely due to: 1. homonymous hemianopsia. 2. astereognosis. 3. unilateral neglect. 4. apraxia.

4. apraxia. 4. Constructional apraxia describes a cognitive dysfunction in which a patient has difficulty initiating and performing a sequence of movements. In this case, the patient is unable to assemble blocks.

A 22-year-old patient is hospitalized awaiting a lung transplant due to cystic fibrosis. The patient's physician is interested in an objective measure of the patient's preoperative endurance. Which of the following tests is MOST appropriate for the physical therapist to administer to this patient? 1. VO2 max treadmill test 2. Two-step exercise test 3. Submaximal exercise test on a cycle ergometer 4. 6-minute walk test

4. 6-minute walk test 1. The VO2 max treadmill test examines functional cardiovascular capacity, not necessarily endurance (Kauffman, pp. 37-38). 2. The two-step exercise test is the clinical standard for assessing exercise-induced ischemia. While taking this test, it is possible to reach VO2 max or near VO2 max, which is not the most appropriate clinical situation for the patient described in the stem. (Ehrman, p. 61) 3. The submaximal exercise test is used to help estimate VO2 max and assessed aerobic power, not endurance (Miller, pp. 114-117). 4. The 6-minute walk test is designed to measure endurance for acutely ill individuals who have cardiovascular and pulmonary dysfunction (Hillegass, pp. 683, 688).

Which of the following techniques is MOST appropriate for treatment of a patient with low postural tone? 1. Slow regular rocking while sitting on a treatment bolster 2. Continuous pressure to the skin overlying the back muscles 3. Low-frequency vibration to the back muscles 4. Joint approximation applied through the shoulders to the trunk

4. Joint approximation applied through the shoulders to the trunk 1. Rhythmic rocking is used to decrease postural tone, which is not indicated for this patient (p. 215). 2. Pressure is used to decrease postural tone, which is not indicated for this patient (p. 214). 3. Vibration is used to decrease postural tone, which is not indicated for this patient (p. 214). 4. Of the options listed, joint approximation is the most appropriate technique for improving low postural tone (pp. 827-828).

A patient with muscular dystrophy was removed from mechanical ventilation 1 day ago. The patient is currently unable to independently clear secretions, despite receiving instruction in the bronchopulmonary hygiene techniques of positioning, percussion, shaking, and vibration. Which of the following interventions is MOST appropriate to help this patient clear the secretions? 1. Nasotracheal suctioning 2. Manual costophrenic assist 3. Supplemental oxygen 4. Inspiratory muscle training

1. Although nasotracheal suctioning is a viable option to clear a patient's secretions, it is usually the last resort when a patient does not have an artificial airway. If a manual costophrenic assist does not work, then suctioning may be needed. 2. Any patient who has been receiving mechanical ventilation will likely have some respiratory muscle deconditioning. With superimposed neuromuscular disease, the respiratory muscle weakness will be further exacerbated. Therefore, providing manual assist at the lower ribs during cough will assist the patient in successfully clearing secretions. 3. Supplemental oxygen is indicated to increase arterial oxyhemoglobin saturation but does not ensure that the patient will be able to generate enough force during the cough. 4. Inspiratory muscle training would be beneficial for this patient, but the removal of secretions is most immediately important. Once secretions are under control, then muscle training can begin.

Acquired immunodeficiency syndrome is MOST likely present in a patient with which of the following hematologic conditions? 1. CD4 count of 150 cells/mL 2. High antinuclear antibody titer 3. High human leukocyte antigen titer 4. Neutrophil count of less than 5000 cells/mm3

1. CD4 count of 150 cells/mL 1. Patients who have acquired immunodeficiency syndrome include those who have human immunodeficiency virus type 1 (HIV-1) and a CD4 count below 200 cells/mL (p. 282). 2. A high antinuclear antibody titer is not specific to the presence of acquired immunodeficiency syndrome (p. 1726). 3. A human leukocyte antigen titer is not specific to the presence of acquired immunodeficiency syndrome (p. 1726). 4. A low neutrophil count is not specific to the presence of acquired immunodeficiency syndrome (p. 1715).

A physical therapist is positioning a patient for postural drainage. To BEST drain the posterior segment of both lower lobes, the patient should be placed in which of the following positions? 1. Prone, head down at a 45° angle 2. Supine, flat surface 3. Sidelying, head elevated at a 30° angle 4. Sitting, leaning forward

1. Prone, head down at a 45° angle 1. The best position for draining the posterior segment of both lower lobes would be prone lying with the head down and the lower extremities and hips elevated to about 45°. 2. Supine on a flat surface is the position used to drain the anterior segments of the upper lobes. 3. Sidelying with the head elevated is not a standard position used in postural drainage. Sidelying with the head down is used to drain the lateral segments of the lower lobes. 4. Sitting while leaning forward is the position used for draining the apical segments of the upper lobes.

A physical therapist researcher is developing a study to compare differences in range of motion outcomes in two groups of patients who have had a total knee arthroplasty. Over an 8-week period, one group of patients receives outpatient physical therapy 1 time/week and the other group receives outpatient physical therapy 3 times/week. In this study, what is the dependent variable? 1. Range of motion 2. Frequency of visits 3. Total knee arthroplasty 4. 8 weeks

1. Range of motion The dependent variable, range of motion, is the factor that is caused by the independent variable, frequency of visits.

To minimize skin irritation during functional electrical nerve stimulation, a physical therapist should use: 1. lower intensity, larger interelectrode distance, and larger electrodes. 2. lower intensity, larger interelectrode distance, and smaller electrodes. 3. higher intensity, smaller interelectrode distance, and smaller electrodes. 4. lower intensity, smaller interelectrode distance, and larger electrodes.

1. lower intensity, larger interelectrode distance, and larger electrodes. 1. Several things can be done to decrease the current density and the possibility of skin irritation. These include decreasing the intensity of the stimulation, increasing the interelectrode distance, and using larger electrodes. 2. Larger electrodes, not smaller, should be used to minimize skin irritation. 3. Higher intensity, smaller interelectrode distance, and smaller electrodes would all increase skin irritation. 4. Larger, not smaller, interelectrode distance should be used to minimize irritation.

In a research study, the independent t-test was used as the statistical tool. How would statistically significant results be presented if alpha was set at .05? 1. p <.05 2. p >.05 3. r2 >.05 4. r2 <.05 Your Answer: 2 Correct Answer: 1 You have incorrectly answered the question.

1. p <.05 1. A p value is the probability value. With the alpha set at .05, p values less than .05 are considered statistically significant. (pp. 272, 287) The r value is the Pearson product-moment correlation coefficient, and r2 is the coefficient of determination (the percentage of variance that is shared by the two variables that are correlated) (pp. 319, 323-324). Statistically significant results would be presented by reporting a p value (

A physical therapist is evaluating the degree of atrophy in a 36-year-old patient who has partial resection of the musculocutaneous nerve. Girth is BEST measured from which of the following landmarks? 1. 2 inches (5 cm) proximal to the radial styloid 2. 3.9 inches (10 cm) proximal to the lateral epicondyle of the humerus 3. 2 inches (5 cm) distal to the radial tuberosity 4. 3.9 inches (10 cm) distal to the medial epicondyle of the humerus

2. 3.9 inches (10 cm) proximal to the lateral epicondyle of the humerus The musculocutaneous nerve innervates the biceps brachii, brachialis, and coracobrachialis muscles, which are all located in the anterior arm, along with sensory innervations to the lateral forearm. Nerve injury can result in weakness and atrophy of the muscles innervated. Among the options presented, 10 cm proximal to the lateral epicondyle of the humerus is the only choice that corresponds to the location of these muscles and is, therefore, the correct response. all other choices: the muscles innervated by the musculocutaneous nerve are located in the arm, not forearm.

Prior to starting an exercise training program, a patient with cardiac problems who is taking beta-blocking medication should receive an explanation of the: 1. greater benefits from cardiovascular exercise to be achieved at lower rather than at higher metabolic levels. 2. need to use measures other than heart rate to determine intensity of exercise. 3. need for exercise training sessions to be more frequent but of shorter duration. 4. need for longer warm-up periods and cool-down periods during exercise sessions.

2. need to use measures other than heart rate to determine intensity of exercise. 1. Even though heart rate and blood pressure would be lower due to the beta-blocker, the patient actually achieves the same metabolic levels during exercise. The patient will not achieve greater benefits from exercise at lower metabolic levels due to taking the medication. 2. A patient taking beta-blocking medication will experience a lower heart rate and blood pressure response during exercise, compared to a patient who is not taking this type of medication. Heart rate is lower than anticipated in patients taking beta-blockers, and using heart rate to monitor exercise intensity may not give an accurate indication of intensity. Another measure, such as the Borg scale (rating of perceived exertion), would be more beneficial. 3. Even though heart rate and blood pressure would be lower due to the beta-blocker, the patient actually achieves the same metabolic levels during exercise. Therefore, altering the frequency or duration of exercise is unnecessary. 4. The time for warm-up and cool-down exercises does not need to be altered.

A manual muscle test of a patient who sustained a gunshot wound immediately superior to the elbow joint reveals specific muscle weakness from a partial median nerve injury. The physical therapy intervention for the patient should include strengthening activities for wrist flexion, forearm: 1. pronation, finger flexion, and thumb (1st digit) adduction. 2. pronation, finger flexion, and thumb (1st digit) opposition. 3. supination, finger abduction, and thumb (1st digit) opposition. 4. supination, finger flexion, and thumb (1st digit) extension.

2. pronation, finger flexion, and thumb (1st digit) opposition. 1. Thumb (1st digit) adduction is accomplished by the adductor pollicis (ulnar nerve). 2. The median nerve innervates the following muscles in the forearm: (1) pronator teres and quadratus, (2) flexor digitorum superficialis, (3) flexor digitorum profundus (index and middle fingers [2nd and 3rd digits]), (4) thenar muscles (abductor pollicis brevis, opponens pollicis, flexor pollicis brevis). Therefore, a lesion of the median nerve would affect those muscles and their accompanying actions: forearm pronation, finger flexion, and thumb (1st digit) opposition.

When treating a patient who has ankylosing spondylitis, which of the following muscles require the MOST emphasis in strengthening exercises? 1. Pectorals 2. Hip flexors 3. Back extensors 4. Abdominals

3. Back extensors 1. Persons who have ankylosing spondylitis tend to assume flexed postures. Disproportionately strong pectorals could worsen the kyphosis. 2. Persons who have ankylosing spondylitis tend to assume flexed postures. The hip flexors are already expected to be in a shortened and tight position; stretching, not strengthening, the hip flexors would be more appropriate. 3. Trunk range of motion exercises and strengthening back extensors to minimize thoracic kyphosis are essential. Persons who have ankylosing spondylitis tend to assume flexed postures. 4. Persons who have ankylosing spondylitis tend to assume flexed postures. Disproportionately strong abdominals could worsen the kyphosis.

A patient with chest pain from myocardial ischemia will MOST likely exhibit: 1. increased pain upon chest-wall palpation. 2. increased pain with deep breathing. 3. relief with nitroglycerin (Nitrostat). 4. relief with antacid.

3. relief with nitroglycerin (Nitrostat). 1. Increased pain with chest-wall palpation is more indicative of a musculoskeletal origin of pain. 2. Increased pain with deep breathing is more indicative of a pulmonary origin of pain. 3. Nitroglycerin (Nitrostat) is a common vasodilator that is prescribed for patients who have angina. A vasodilator will improve myocardial blood flow and help relieve ischemia and its manifestations. 4. Relief of pain with antacid ingestion is more indicative of referred pain from peptic ulcer disease.

Following trauma at the C5 spinal cord level, a patient was admitted to the hospital. Twenty-four hours later, the patient shows no reflexes, sensation, or voluntary motor activity below the level of injury. These findings indicate: 1. the presence of spasticity. 2. decerebrate rigidity. 3. spinal shock. 4. a lower motor neuron lesion.

3. spinal shock. 1. Spasticity is associated with hyperreflexia and increased muscle tone. Spasticity would be expected to develop following the spinal shock stage. 2. Decerebrate rigidity involves a sustained contraction of the upper and lower extremities in extension. 3. Spinal shock occurs as a reaction to spinal cord injury and is characterized by an absence of all reflex activity below the level of the lesion. Depending on the extent of the lesion, the patient may lose all or some sensation and motor activity below the level of the lesion. 4. With a lower motor neuron lesion, the loss of sensation and motor activity would be confined to a much smaller region (depending on the exact lesion) and would not affect all levels below the lesion.

A patient with idiopathic pulmonary fibrosis completed a 6-minute walk test and demonstrates the following results: total walking distance of 1200 ft (366 m) in 6 minutes, heart rate of 82 to 110 bpm (pretest to posttest), blood pressure of 125/80 to 145/85 mm Hg (pretest to posttest), respiratory rate of 18 to 40 breaths/minute (pretest to posttest), and oxygen saturation of 98% to 92% (pretest to posttest); an electrocardiogram showed normal sinus rhythm throughout the test. Based on these results, the physical therapist should determine that the patient has impaired: 1. aerobic capacity and endurance associated with cardiovascular pump dysfunction. 2. ventilation, respiration, and aerobic capacity associated with airway clearance dysfunction. 3. ventilation, respiration, aerobic capacity, and gas exchange associated with ventilatory pump dysfunction. 4. aerobic capacity and endurance associated with myocardial ischemia.

3. ventilation, respiration, aerobic capacity, and gas exchange associated with ventilatory pump dysfunction. 1. Based on the walk test results, the heart rate and blood pressure have normal physiologic rise in response to exercise and would not indicate cardiovascular pump dysfunction. 2. Although the walk test results do indicate impaired ventilation and respiration, there is no indication of airway clearance issues in the question. 3. In general, a patient who has pulmonary fibrosis will have an impaired ventilatory pump. This is further evidenced by the exaggerated respiratory rate response and desaturation in the 6-minute walk test results. 4. The normal electrocardiogram does not suggest myocardial ischemia.

A 78-year-old patient who is being treated for osteoarthritis of the knees reports centralized lower thoracic pain and epigastric pain. The pain is relieved by eating. Which of the following steps would be MOST important in screening for the cause of the new symptoms? 1. Resist the iliopsoas muscle to screen for a psoas abscess. 2. Ask if the patient has been constipated or has had diarrhea. 3. Perform an abdominal examination to screen for an abdominal aortic aneurysm. 4. Ask if the patient is taking a high dose of nonsteroidal anti-inflammatory drugs.

4. Ask if the patient is taking a high dose of nonsteroidal anti-inflammatory drugs. 1. A psoas abscess would be painful in the right or left lower quadrant and refer pain to the low back. 2. Constipation and diarrhea are symptoms related to the colon, which, when painful, relates to mid-abdominal pain and refers pain to the sacral area. 3. An abdominal aortic aneurysm would likely cause severe low back pain and would not change with eating nor cause epigastric pain. 4. A high percentage of hospitalizations of the aging population who have gastrointestinal problems are due to the effects of nonsteroidal anti-inflammatory drugs. This patient may be taking this class of drugs for the pain and inflammation in the knees. Because the pain changes with food intake, the gastric region as a source is implicated.

A 15-month-old patient who has clubfoot will MOST likely hold the foot and ankle in which of the following positions? 1. Dorsiflexion and calcaneal varus 2. Plantar flexion and calcaneal valgus 3. Dorsiflexion and forefoot abduction 4. Plantar flexion and forefoot adduction

4. Plantar flexion and forefoot adduction 4. Talipes equinovarus, or clubfoot, involves plantar flexion, forefoot adduction, and calcaneal varus.

A physical therapist reviews the chart of an inpatient and finds that the patient's angiogram shows a complete blockage of the left middle cerebral artery. Which of the following conditions is the therapist MOST likely to note upon performing an evaluation? 1. Hemiparesis in the left upper extremity 2. Presence of hemineglect syndrome 3. Left homonymous hemianopsia 4. Presence of expressive aphasia

4. Presence of expressive aphasia 1. Left hemiparesis occurs with a blockage of the right middle cerebral artery, not the left middle cerebral artery. 2. Hemineglect syndrome is much more common in right cerebral hemisphere lesions than in left hemisphere lesions. 3. Left homonymous hemianopsia occurs with a blockage of the right middle cerebral artery, not the left middle cerebral artery. 4. Complete blockage of the left middle cerebral artery would most likely cause a problem with expressive speech (Broca aphasia), especially in a right-handed individual. Infarction of the dominant hemisphere can lead to global aphasia.

To stretch the neck muscles in a patient with an acute, right-sided torticollis, the MOST appropriate therapeutic exercise is: 1. right rotation and right lateral flexion. 2. left rotation and right lateral flexion. 3. left rotation and left lateral flexion. 4. right rotation and left lateral flexion.

4. right rotation and left lateral flexion. 4. In right-sided torticollis, isolated contraction of the right sternocleidomastoid muscle would cause the head to rotate to the left and side bend (lateral flexion) to the right. To stretch the muscle, the opposite actions would be performed on the patient by the therapist: rotation of the head to the right and lateral flexion to the left.

A physical therapist is interviewing a patient who awoke that day with a slight loss of taste sensation, inability to close the right eye, and dryness of the right eye. The patient has had a slight headache the last few days and some mild pain around the right ear. Which of the following questions should the therapist ask the patient to identify an additional finding that is consistent with these signs and symptoms? 1. Are you able to smile? 2. Do you have pain over your right jaw? 3. Are you able to chew food? 4. Do you have difficulty swallowing?

1. Are you able to smile? 1. All of the questions are important in history taking; however, the suspected diagnosis is Bell palsy caused by a lesion of the facial nerve (CN VII). The patient would most likely have weakness in the muscles of facial expression, including smiling. Asking the patient to smile would most likely show an asymmetrical smile with weakness on the right side. 2. Jaw pain would be more likely due to a lesion of the trigeminal nerve (CN V). 3. Inability to chew food would be more likely due to a lesion of the trigeminal nerve (CN V). 4. Inability to swallow may be caused by damage to the glossopharyngeal (CN IX), vagus (CN X), or hypoglossal (CN XII) nerves.

To relieve lateral shoulder and arm pain, a patient elevates the right arm with the elbow bent and rests the right forearm on the head. The patient's symptoms are MOST likely caused by which of the following conditions? 1. C4-C5 disc herniation 2. Rotator cuff tear 3. Thoracic outlet syndrome 4. Anterior shoulder instability

1. C4-C5 disc herniation 1. The shoulder abduction test described in the question evaluates for radicular symptoms and involves the C4 or C5 nerve roots. If resting the arm on the head relieves the patient's symptoms, a cervical compression problem or nerve root compression should be suspected. Lateral shoulder pain occurs in the location of the C4 dermatome. (p. 1297) 2. The rotator cuff would be evaluated using tests such as the drop arm or lift off test. Upper extremity elevation is likely to be limited by pain in a patient who has a rotator cuff tear. The shoulder abduction test described in the question would likely elicit pain, not relieve it. (p. 637) 3. The shoulder abduction test described in the question evaluates for the presence of radicular symptoms and involves the C4 or C5 nerve roots. Thoracic outlet syndrome would be evaluated using tests, such as the Roos test. Upper extremity elevation is more likely to exacerbate thoracic outlet syndrome symptoms, not relieve them. (p. 1300) 4. Anterior shoulder instability would be evaluated using the apprehension test, not the shoulder abduction test described in the question, which is used to evaluate radicular symptoms. Shoulder abduction and lateral (external) rotation would exacerbate symptoms in a patient with anterior shoulder instability

Which of the following methods is MOST appropriate for handling a 1-year-old child with cerebral palsy who exhibits strong extensor tone in the trunk and extremities? 1. Carrying the child in a sitting position 2. Carrying the child over one's shoulder 3. Keeping contact with the back of the child's head 4. Picking the child up under the upper extremities

1. Carrying the child in a sitting position 1. The sitting position promotes visual attending, use of the upper extremities, and social interaction (Martin, pp. 98-100). A flexed posture is preferred so the shoulders are forward. A child who exhibits extensor posturing should be carried in a symmetric position that does not allow axial hyperextension and keeps the hips and knees flexed. (Palisano, p. 465) 2. Carrying the child over one's shoulder would inhibit visual attending and social interaction. It is better to carry the child in such a way that allows the child to look around and see what is ahead. (Martin, p. 101) 3. Carrying positions should accentuate the strengths of the infant and should avoid as much abnormal posturing as possible. The infant should be allowed to control as much of his or her body as possible for as long as possible before external support is given (Martin, p. 148). Contact to the back of the child's head may facilitate extensor posturing. When carrying a child, one should encourage as much head and trunk control as the child can demonstrate. The child should be carried so that the neck and trunk muscles are used to maintain the head and trunk upright against gravity. This allows the child to look around and see what is ahead. (Martin, p. 101) 4. Picking the child up under the upper extremities would be more likely to facilitate extensor posturing. The legs stiffen into extension and may even cross or scissor. (Martin, pp. 98, 100)

When examining a patient with right facial weakness, a physical therapist notes the presence of a right Bell phenomenon. Which of the following additional findings is MOST likely to be present? 1. Decreased taste sensation on the tongue 2. Abnormal tone of the right limbs 3. Ptosis of the right eyelid 4. Loss of pain and temperature sensation across the lower right jaw

1. Decreased taste sensation on the tongue 1. Bell phenomenon is observed with complete interruption of the facial nerve (CN VII) at the stylomastoid foramen. The facial nerve (CN VII) while primarily motor, does carry some sensations, particularly taste sensation from the anterior two-thirds of the tongue. (pp. 1395-1396) 2. Abnormal tone is associated with a supranuclear (central) lesion. Supranuclear lesions produce contralateral voluntary lower facial paralysis, so weakness would be on the other side. A Bell phenomenon is not present with central lesions. (pp. 1395-1396) 3. Ptosis or sagging of the eyelid occurs with weakness of the levator palpebrae muscle. The levator palpebrae muscle is innervated by the oculomotor nerve (CN III). (p. 283) 4. Pain and temperature sensation from the face (jaw) is carried by the trigeminal nerve (CN V) (pp. 1392-1393).

While working in a private practice clinic, a physical therapist observes a patient fall in the parking lot outside the office. The patient sustains a severe laceration to the forearm. The therapist secures a pressure dressing to the wound site but notes that blood is soaking through the dressing and the bandage. Which of the following actions should the therapist perform NEXT? 1. Elevate the limb and apply pressure to the wound. 2. Remove the dressing and bandage and start over with a tighter bandage. 3. Apply additional dressings and bandages and apply pressure to the radial artery. 4. Call the patient's physician and arrange transportation for medical care.

1. Elevate the limb and apply pressure to the wound. 1. Elevation and pressure is the correct next step in controlling bleeding. 2. Applying a tighter bandage may create a tourniquet effect, which is not desirable. 3. Elevation and pressure should be applied first, before additional dressings and bandages. If bleeding appears to be arterial, pressure should be applied to the brachial artery, not the radial artery. 4. Although calling the patient's physician to arrange transportation for medical care may be necessary, it does not immediately control the bleeding.

A patient with early symptomatic human immunodeficiency virus infection is MOST likely to report which of the following symptoms? 1. Fatigue 2. Blurred vision 3. Easy bruising 4. Poor wound healing

1. Fatigue 1. Fatigue is an early symptom of human immunodeficiency virus (HIV) infection. 2. Blurred vision is not associated with early human immunodeficiency virus (HIV) infection. 3. Easy bruising is found at more advanced stages of the disease. 4. Poor wound healing is found at more advanced stages of the disease.

A physical therapist examining a patient's joint play finds restriction in the direction indicated by the arrow in the photograph. To address the restriction, the therapist should include an intervention to increase which motion of the index finger (2nd digit)? 1. Flexion 2. Extension 3. Abduction 4. Rotation

1. Flexion 1. The therapist is shown performing a volar glide, which is the same joint motion used for finger flexion. Limited motion in this direction indicates limited ability to perform finger flexion.

A patient comes to physical therapy with a diagnosis of a medial meniscus tear of the right knee. Which of the following signs and symptoms is MOST indicative of this diagnosis? 1. Mechanical locking 2. Decreased pain with weight-bearing 3. Posterior knee swelling 4. Atrophy of hamstrings

1. Mechanical locking 1. A history of mechanical locking is a common symptom of a knee medial meniscus tear. 2. With a medial knee meniscus tear, pain is commonly increased, not decreased, with weight-bearing. 3. With a medial knee meniscus tear, swelling would more likely be evident anteriorly, not posteriorly. 4. With a medial knee meniscus tear, quadriceps atrophy is more likely than hamstrings atrophy.

A physical therapist is examining a patient for possible lower extremity weakness. Passive range of motion is within normal limits. The patient is seated. When the patient tries to dorsiflex and invert the right foot, the patient is unable to move it through the full range of motion and is unable to resist any force applied by the therapist. During the subsequent gait examination, the therapist should expect the patient to display which of the following gait patterns? 1. Normal gait with no deviations 2. Increased right hip flexion during the midswing phase 3. Backward lean of the trunk on the right side shortly after heel strike (initial contact) 4. Lateral bending toward the right side during midstance

1. Gait deviations are expected when the tibialis anterior demonstrates weakness (p. 308). 2. The tibialis anterior showed Poor (2/5) grade strength during the examination. Because this muscle is active during the swing phase of gait and shortly after heel strike (initial contact), one should expect that gait deviations would be most apparent at those times. Specifically, the tibialis anterior maintains dorsiflexion of the foot during the swing phase and controls plantar flexion after heel strike (initial contact). During the swing phase, weakness of the tibialis anterior would cause the patient to increase the amount of hip and knee flexion to prevent toe drag. (p. 308) 3. Backward lean of the trunk after heel strike (initial contact) may be due to weakness of the hip extensors (gluteus maximus) (p. 307). 4. With weak hip abductors, the patient would be expected to have excessive pelvic rotation. To compensate, the patient would laterally bend toward the weak (right) side during midstance to help prevent the excessive hip drop. (p. 307)

While bending over in standing position with the knees extended, a patient exhibits decreased spinal flexion and decreased hip flexion. The findings MOST likely are associated with limitation in which of the following measures? 1. Hamstring flexibility 2. Gluteal muscle strength 3. Abdominal muscle strength 4. Hip flexor flexibility

1. Hamstring flexibility 1. Tightness of the hamstrings causes excessive lumbar flexion, because tight hamstrings restrict the amount of hip flexion.

A patient has sustained a traumatic brain injury and is slow to respond to all stimuli. The patient currently verbalizes and moves away from noxious stimuli and is able to close the eyes 3 of 5 times when asked. What level on the Rancho Los Amigos levels of cognitive functioning scale BEST describes this patient's current status? 1. Localized response 2. Confused-agitated 3. Generalized response 4. Confused-inappropriate

1. Localized response 1. The patient described in the question is functioning at the localized response level (level III) of the Rancho Los Amigos levels of cognitive functioning scale. Localized response is defined as a patient having a localized response and reacting specifically but inconsistently to stimuli. Responses are delayed and directly related to the type of stimulus present. 2. Confused-agitated response (level IV) is defined as the state where a patient is at a heightened state of activity. 3. Generalized response (level II) is defined as a generalized response where a patient reacts non-purposefully to stimuli. 4. Confused-inappropriate response (level V) occurs when the patient is able to respond to simple commands fairly consistently.

In the INITIAL management of a patient with a partial peripheral nerve injury, the goal of the physical therapy intervention will MOST likely be to prevent which of the following problems? 1. Nerve degeneration 2. Spasticity and clonus 3. Muscle atrophy 4. Contractures and adhesions

1. Nerve degeneration is not affected by therapy; it is determined by the extent of the injury (p. 387). 2. Peripheral nerve injuries cause weakness of muscles and flaccidity, not increased muscle tone (p. 376). 3. When there are signs of reinnervation, motor retraining can begin to rebuild muscle, but not in the initial phase (p. 390). 4. The primary physical therapy goal in this case would be joint protection and the prevention of contractures and adhesions, usually through splinting (p. 389).

A physical therapist is examining the integumentary system of a patient with light skin pigmentation and notices a deepening of the skin color over the left posterior aspect of the calcaneus. Which of the following findings would indicate a Stage 1 pressure injury at that site? 1. Nonblanchable redness 2. Blanching with applied pressure that returns to previous coloring within 10 seconds after the removal of pressure 3. Blanching with applied pressure that returns to previous coloring within 30 seconds after the removal of pressure 4. Blanching with applied pressure that returns to previous coloring 1 hour after the removal of pressure

1. Nonblanchable redness 1. The onset of a Stage 1 pressure injury is characterized by nonblanching erythema.

Which of the following examination findings is consistent with dehydration? 1. Poor turgor 2. Dependent edema 3. Pitting edema 4. Hypertension

1. Poor turgor 1. Poor skin turgor is one of the signs of dehydration. The skin, when lifted up between the fingers, does not return to its original position in a rapid manner. 2. Dependent edema and pitting edema are findings of volume excess, the opposite state of dehydration. 3. Dependent edema and pitting edema are findings of volume excess, the opposite state of dehydration. 4. Dehydration is associated with postural hypotension, not hypertensio

A physical therapist is providing intervention to a patient with lower extremity peripheral neuropathy who is confined to bed. When providing education to the patient and caregivers, which of the following suggestions should the therapist emphasize? 1. Position foam cushions under the calves. 2. Visually inspect the skin every other day. 3. Place a soft pillow under the heels. 4. Change the patient's position every 4 hours.

1. Position foam cushions under the calves. Rationales: 1. Cutaneous areas lacking sensation are at the highest risk for skin breakdown from trauma or pressure. Heels should be elevated off the supporting surfaces. (p. 267) 2. Visual inspection should occur daily and throughout the day (p. 310). 3. Individuals with distal neuropathy are at a great risk for development of ulcers over weight-bearing surfaces of the feet. Heel pressure on all surfaces should be minimized. (p. 267) 4. Positioning is dependent on the specific condition of each individual patient. Positioning requires relieving pressure from bony prominences, which is more important than changing positions at timed intervals. (p. 267)

A male patient has been referred to physical therapy with lumbar pain that radiates around the iliac crest to the groin. Physical examination reveals pain unchanged by position, no point tenderness in the paraspinal muscles, a positive iliopsoas sign, and supraclavicular adenopathy. Which of the following actions should the physical therapist take FIRST? 1. Refer the patient back to the physician for suspicion of testicular cancer. 2. Refer the patient back to the physician for suspicion of herniated nucleus pulposus. 3. Administer tests of lower extremity sensation and strength. 4. Administer tests of neurodynamic mobility and trunk range of motion.

1. Refer the patient back to the physician for suspicion of testicular cancer. 1. The examination results, especially the supraclavicular adenopathy and positive iliopsoas sign, are indicative of testicular cancer, not a herniated nucleus pulposus. 2. The patient's back pain is suspicious for testicular cancer and is not consistent with mechanical back pain caused by a herniated nucleus pulposus.

A patient has a history of a mild stroke, type 2 diabetes, hypertension, and impaired balance. During examination, the patient performs the Clinical Tests for Sensory Integration and Balance. The patient demonstrates increased sway and difficulty maintaining balance when wearing a dome while standing on a firm surface and when wearing a dome while standing on a foam surface. Based on the findings, which of the following systems or system interactions is the patient MOST dependent upon for balance? 1. Visual 2. Vestibular 3. Somatosensory 4. Sensory selection

1. Visual 1. Patients who show an increased amount of sway when blindfolded and standing on a firm surface, when wearing a dome and standing on a firm surface, when blindfolded and standing on a foam surface, and when wearing a dome and standing on a foam surface, are thought to be highly dependent upon vision for balance. 2. Patients who have increased sway when standing on foam while wearing a dome or blindfolded are thought to have the inability to select vestibular input for balance. 3. Patients who have increased sway during all conditions standing on foam are thought to be dependent upon somatosensory information. 4. Patients who are unable to adapt sensory information for postural control are described as having a sensory selection problem.

If a transtibial prosthesis has an excessively firm heel, the patient is MOST likely to walk with: 1. excessive knee flexion in foot flat (loading response). 2. premature knee flexion in heel off (terminal stance). 3. insufficient knee flexion in foot flat (loading response). 4. excessive knee flexion in heel off (terminal stance).

1. excessive knee flexion in foot flat (loading response). 1. Upon heel contact, the heel section compresses, partially absorbing the ground impact and thus permitting a controlled plantar flexion. If the heel is too stiff or is too high, a flexion moment will occur at the knee. This flexion moment reduces stance stability and increases the likelihood of the knee buckling in foot flat (loading response) and midstance. 2. An excessively firm heel would not be a factor in heel off (terminal stance). 3. Excessive knee flexion is expected. 4. An excessively firm heel would not be a factor in heel off (terminal stance).

The physical therapist for a college track team is evaluating a hurdler. One week earlier, the patient had to stop suddenly during a race because of sharp, severe buttock pain. The patient now reports pain with straight-leg raising and resisted knee flexion, and swelling in the buttock. The MOST likely cause of this patient's symptoms is an avulsion fracture at the: 1. ischial tuberosity. 2. inferior pubic ramus. 3. lesser trochanter. 4. anterior inferior iliac spine.

1. ischial tuberosity. 1. An avulsion fracture of the ischial tuberosity is usually reported with an acute traumatic incident. The pain is usually local, but may also radiate down the thigh. Active or resisted knee flexion would increase the pain. 2. Inferior pubic ramus fractures are associated with a gradual onset of groin pain that increases with weight-bearing and walking. 3. Fractures in the trochanter region are associated with groin pain and tenderness in the area. 4. Anterior inferior iliac spine fractures would not be associated with buttock pain or pain with resisted knee flexion, because the muscles that attach at these locations are the knee extensors and hip flexors.

A physical therapist is treating a patient with moderately controlled type 2 diabetes. One of the goals is to help the patient regulate insulin dosage through exercise. In establishing an exercise program, the therapist should be aware that regular exercise will generally: 1. lower blood glucose levels and decrease the amount of insulin required. 2. lower blood glucose levels and increase the amount of insulin required. 3. raise blood glucose levels and decrease the amount of insulin required. 4. raise blood glucose levels and increase the amount of insulin required.

1. lower blood glucose levels and decrease the amount of insulin required. 1. In patients with moderate hyperglycemia, exercise can lead to hypoglycemia for periods of 24 to 48 hours after exercise. Exercise has been shown to increase sensitivity of the insulin receptors, leading to a decrease in the amount of insulin required.

During evaluation of a patient's gait, a physical therapist observes that the patient leans forward shortly after heel strike (initial contact). The patient's forward bending is MOST likely a compensation for weakness in the: 1. quadriceps. 2. hamstrings. 3. gluteus maximus. 4. tibialis anterior.

1. quadriceps. 1. The quadriceps are active shortly after heel strike (initial contact) to prevent excessive knee bending during the loading phase of initial stance. Weakness of the quadriceps causes the patient to compensate by leaning forward at heel strike (initial contact) and to use the body weight to help keep the knee extended. (p. 695) 2. Weakness of the hamstrings causes excessive knee extension (recurvatum) prior to heel strike (initial contact) (p. 693). 3. Weakness of the gluteus maximus would be observed shortly after heel strike (initial contact). However, the patient would lean backward to compensate. (p. 694) 4. Weakness of the tibialis anterior causes a "foot slap" just after heel strike (initial contact) (p. 691).

A physical therapist is working with a patient who has multiple medical issues and has just finished chemotherapy. Which of the following tests is MOST appropriate to measure changes in this patient's endurance over time? 1. 10-meter walk for time 2. 6-minute walk 3. Timed Up and Go 4. Maximum VO2 assessment

2. 6-minute walk 1. The 10-meter walk for time test examines gait speed, not necessarily endurance (Kauffman, p. 493). 2. The 6-minute walk test is used to examine endurance (Hillegass, pp. 683, 688). 3. The Timed Up and Go test is used to assess fall risk, not endurance (Hillegass, p. 690). 4. A maximum VO2 is the maximum rate of oxygen consumption as measured during incremental exercise and does not necessarily indicate endurance (Kauffman, pp. 37-38).

During an initial evaluation, which of the following tests is MOST likely to identify an abnormal finding for a patient who has acute right-sided heart failure? 1. Sensory testing of the upper extremities 2. Circumferential girth measurement of the lower extremities 3. Resisted manual muscle testing of all extremities 4. Peripheral pulse testing of the lower extremities

2. Circumferential girth measurement of the lower extremities Right-sided heart failure results in dependent edema. Circumferential girth measurements of the lower extremities to monitor edema are appropriate to determine the severity of right-sided heart failure and aid the therapist in treatment planning.

When performing manual lymphatic drainage, which of the following techniques is MOST appropriate to use? 1. Decongest the distal segments before progressing to the proximal segments. 2. Decongest the involved trunk quadrant before decongesting the involved limb. 3. Decongest proximal segments before decongesting the involved trunk quadrant. 4. Decongest distal portions of the involved limb before decongesting the involved trunk quadrant.

2. Decongest the involved trunk quadrant before decongesting the involved limb. Lymphatic drainage is performed with the goal of directing the lymphatics centrally. If the therapist does not first decongest the trunk quadrants, the lymphatics will remain blocked. SO trunk first, followed by proximal to distal segments

Which of the following structures is indicated by the arrow in the radiograph? 1. Intertubercular groove 2. Greater tubercle 3. Lesser tubercle 4. Coracoid process

2. Greater tubercle 1. The intertubercular (bicipital) groove is located just to the right of the position indicated by the arrow. 2. The structure indicated by the arrow is the greater tubercle. The arm is medially (internally) rotated in this radiograph, as evidenced by the position of the bicipital groove. 3. The lesser tubercle is located inferiorly and to the right of the position indicated by the arrow. 4. The coracoid process is located superiorly and to the right of the position indicated by the arrow

An exercise program following recent closed reduction of an anterior dislocation of the shoulder should AVOID which of the following motions? 1. Adduction and medial (internal) rotation 2. Hyperextension and lateral (external) rotation 3. Abduction and flexion 4. Extension and medial (internal) rotation

2. Hyperextension and lateral (external) rotation With hyperextension and lateral (external) rotation, the humeral head must glide anteriorly due to the convex-concave rule. Both of these motions put additional stress on the anterior capsule via the position of the humeral head during exercise and should be avoided.

Which of the following modalities BEST addresses calcific tendinitis in the bicipital tendon? 1. Sensory level interferential current 2. Iontophoresis with acetic acid 3. High-voltage pulsed electrical stimulation 4. Diathermy with a parallel treatment set-up

2. Iontophoresis with acetic acid 1. Sensory level interferential current does not address the problem itself but may address any related pain (Bellew, p. 346; Cameron, pp. 224-225, 231). 2. Iontophoresis with acetic acid can address calcific tendinitis, not just the symptoms (Bellew, p. 319; Cameron, p. 274). 3. High-voltage pulsed electrical stimulation does not address the problem itself but may address any related pain (Bellew, p. 275; Cameron, pp. 224-226, 231). 4. Diathermy with a parallel treatment set-up is not the best choice, because deep heat will not address the problem (Bellew, p. 186; Cameron, p. 208-209).

A 20-year-old female athlete underwent anterior cruciate ligament reconstruction 3 days ago. Which of the following exercises is LEAST indicated at this time? 1. Performing active knee flexion to 90° in prone position 2. Kicking a large air-filled ball from a short-sitting position 3. Performing quadriceps isometrics in a long-sitting position 4. Hip abduction with a 1-lb (0.5-kg) weight at the ankle in sidelying position

2. Kicking a large air-filled ball from a short-sitting position 2. One of the key components of the immediate postoperative phase following anterior cruciate ligament reconstruction is protection of the graft by minimizing strain on the anterior cruciate ligament. Open-chain knee extension is avoided during this time period because it causes significant stress on the ligament and joint.

A patient who has a history of receiving radiation therapy reports a sensation of fullness and tightness of the forearm, limited wrist and finger movements, and difficulty wearing a wristwatch and ring. Which of the following tests and measures is MOST appropriate to perform? 1. Check for pulse. 2. Measure limb girth. 3. Obtain blood pressure. 4. Grasp skin on dorsum of hand.

2. Measure limb girth. 1. The clinical manifestations of swelling and limited range of motion combined with the patient's ongoing treatment with radiation are suggestive of lymphedema. Arterial circulatory compromise is not suspected, and, therefore, taking a pulse would not be the most useful test. 2. Radiation therapy is a risk factor for lymphedema. A sensation of fullness and tightness of skin, decreased flexibility, and difficulty wearing clothes and accessories are clinical manifestations of lymphedema. Accurate circumferential measurements taken at fixed intervals with standardized positioning noted are often more helpful for describing the limb segments than volumetric measurements. To measure girth of the upper extremity, the appropriate landmark is selected, and the limb circumference at that landmark and at 3.94 inch (10 cm) intervals to the axilla (or along the extent of the edema) is measured and recorded. 3. The clinical manifestations of swelling and limited range of motion combined with the patient's ongoing treatment with radiation are suggestive of lymphedema. Peripheral arterial disease or hypertension are not suspected, and, therefore, taking blood pressure would not be the most useful test. 4. The clinical manifestations of swelling and limited range of motion combined with the patient's ongoing treatment with radiation are suggestive of lymphedema. Dehydration is not suspected, and, therefore, checking skin turgor by grasping the skin on the dorsum of the hand would not be the most useful test.

Which of the following nerves innervates the pelvic floor muscles? 1. Sciatic 2. Pudendal 3. Inferior gluteal 4. Genitofemoral

2. Pudendal 1. The sciatic nerve contains nerve roots from L4-S3 and innervates multiple muscles in the lower leg. 2. The pelvic floor muscles are innervated by sacral nerve roots S2-S4 through the pudendal nerve. 3. The inferior gluteal nerve is a motor nerve that innervates the gluteus maximus. 4. The genitofemoral nerve, which is from L1-L2, is a cutaneous nerve that is sensory and not mot

A patient with type 2 diabetes is evaluated by a physical therapist and found to be alert and oriented to person, place, and time. When continuing the evaluation, the therapist should give the HIGHEST priority to which of the following systems? 1. Cognitive 2. Sensory 3. Genitourinary 4. Musculoskeletal

2. Sensory 2. Type 2 diabetes can result in dysfunctions in the sensory system and pose significant risk to the patient's skin integrity. Given that the patient is oriented to person, place, and time, the therapist can assume that the patient would inform someone in the case of developing pain over a bony prominence (a common site of decubitus ulcers), given an intact sensory system

A physical therapist is preparing to teach a patient with Guillain-Barré syndrome to transfer from wheelchair to mat table. The patient weighs 150 lb (68 kg) and has Poor (2/5) strength in both of the lower extremities and Fair (3/5) strength in the left upper extremity. The right upper extremity has Normal (5/5) strength. Which of the following assisted transfers is MOST appropriate for the patient's initial session? 1. Two-person lift to the right side 2. Sliding-board transfer to the right side 3. Hydraulic lift from wheelchair to mat 4. Full standing-pivot transfer to the right side

2. Sliding-board transfer to the right side 1. Since the patient does have fair to normal upper extremity strength and is not totally dependent, a two-person lift would not be appropriate because it would not enhance the patient's functional skills. 2. The most appropriate transfer method would be for the patient to use a sliding board transfer to the strongest (right) side because the patient does have fair to normal upper extremity strength and is not totally dependent. 3. Since the patient does have fair to normal upper extremity strength and is not totally dependent, a hydraulic lift would not be appropriate because it would not enhance the patient's functional skills. 4. Because the patient has significant weakness in both lower extremities, a standing-pivot transfer would not be safe.

A physical therapist is working with an outpatient who had a cerebrovascular accident and currently lives in an assisted-living facility. Which of the following statements is the MOST appropriate functional goal for this patient? 1. The patient will be able to don an ankle-foot orthosis with assistance. 2. The patient will independently walk 165 ft (50 m) with a straight cane from the bedroom to the cafeteria. 3. The patient will have Normal (5/5) strength of the quadriceps muscles. 4. The patient's balance will improve to be able to independently stand on the involved lower extremity for 20 seconds.

2. The patient will independently walk 165 ft (50 m) with a straight cane from the bedroom to the cafeteria. A goal should have four elements: audience; behavior that is linked to function, activity, or participation; condition that includes situation, position of patient; and degree, which is the amount of assistance. This option has clearly defined elements that are objective, measurable, and functional.

During the shoulder examination of a patient, a physical therapist notes the presence of a capsular pattern without radicular pain. To help establish the cause of the capsular pattern, the therapist should NEXT: 1. perform axial compression on the cervical spine to check for nerve root compression. 2. ask the patient if there has been any prior trauma to the shoulder joint. 3. check for a painful arc during active range of motion. 4. examine the shoulder for a rotator cuff tear.

2. ask the patient if there has been any prior trauma to the shoulder joint. 1. A nerve root impingement commonly results in radicular pain. The patient does not have radicular pain. 2. One of the most common causes of a capsular pattern in the shoulder is traumatic arthritis following injury to the shoulder. The capsular pattern may gradually develop over time. 3. A painful arc is often associated with a noncapsular pattern and may be observed in a patient who has bursitis or tendinitis at the shoulder. The patient described in the scenario has a capsular pattern at the shoulder. 4. A torn rotator cuff is associated with weakness and loss of active shoulder elevation. A capsular pattern results in greater loss of lateral (external) rotation, abduction, and then medial (internal) rotation.

A physical therapist is gait training a patient who suddenly becomes weak and falls to the floor. The patient is now unresponsive. The therapist should FIRST: 1. open the patient's airway. 2. activate the emergency response system. 3. quickly start breathing assistance. 4. begin chest compressions.

2. activate the emergency response system. Because the physical therapist witnessed the patient become unresponsive, it is most appropriate to activate the emergency response system before beginning chest compressions. Chest compressions would only be delivered if the patient does not have a pulse.

A patient with a hiatal hernia is receiving physical therapy. Which of the following exercises would MOST likely worsen the symptoms related to the hernia? 1. Wall sits 2. Overhead press 3. Bilateral leg lifts 4. Hamstring stretch

3. Bilateral leg lifts 1. Wall sits are performed in an upright position and would not exacerbate a hiatal hernia. 2. Overhead press is typically performed in seated, semireclined, or standing position and, therefore, would not exacerbate a hiatal hernia. 3. Individuals who have a hiatal hernia should avoid supine position and avoid the Valsalva maneuver. Bilateral leg lifts must be performed in supine position and require strong contractions of the stomach muscles, encouraging the Valsalva maneuver, which would worsen the hiatal hernia. 4. Hamstring stretching can be modified to be done in a position other than supine to avoid exacerbating a hiatal hernia.

Which of the following factors is considered to be a primary risk factor for atherosclerosis? 1. Stress 2. Obesity 3. Cigarette smoking 4. Sedentary lifestyle

3. Cigarette smoking 1. Stress is a secondary risk factor. 2. Obesity is a secondary risk factor. 3. High blood pressure, cigarette smoking, and hyperlipidemia are direct or primary risk factors for atherosclerosis. 4. Activity level is a secondary risk factor.

A physical therapist is teaching a motor skill to a patient with chronic hemiplegia. Which of the following teaching approaches should be MOST emphasized? 1. Habituation 2. Sensitization 3. Compensatory strategy 4. Recovery of normal movement

3. Compensatory strategy 1. Habituation is a decrease in responsiveness that occurs as a result of repeated exposure to a nonpainful stimulus. 2. Sensitization is an increased responsiveness following a threatening or noxious stimulus. 3. A patient who has chronic hemiplegia is unlikely to recover normal function. Compensatory strategies are used when there is a permanent loss of function that prevents reacquiring normal movement patterns. 4. In a patient who has an acute condition, the emphasis is on recovery of normal function. This patient has a chronic condition. Recovery of normal movement is not a realistic expectation.

What skin change associated with aging has the GREATEST effect on wound healing? 1. Reduction in sensation 2. Decreased elasticity of the skin 3. Decreased epidermal proliferation 4. Change in pigmentation

3. Decreased epidermal proliferation 1. Changes in sensation due to aging may predispose a patient to ulceration but is not a cause of poor wound healing. 2. Changes in elasticity due to aging do not affect wound healing. 3. Wounds heal via a complex process involving re-epithelialization. With advanced aging, the rate of epidermal proliferation decreases. 4. Changes in skin pigmentation due to aging do not affect wound healing.

A patient is being evaluated for possible carpal tunnel syndrome, and a nerve conduction velocity test is performed. Which of the following findings would MOST strongly support the diagnosis? 1. Decreased latency at the elbow 2. Decreased latency at the carpal tunnel 3. Increased latency at the carpal tunnel 4. Increased latency at the forearm

3. Increased latency at the carpal tunnel 1. Nerve conduction above and below the local nerve compression is usually normal in a patient who has carpal tunnel syndrome. Therefore, latency at the elbow should be normal, not decreased. 2. Latency is typically increased, not decreased, across the carpal tunnel compression site. 3. Latency is typically increased across the carpal tunnel compression site. 4. Nerve conduction above and below the local nerve compression is usually normal in a patient who has carpal tunnel syndrome. Therefore, latency at the forearm should be normal, not increased.

An inpatient physical therapy department has only one physical therapist and one physical therapist assistant on duty, due to staff illness. Treatment of a patient with which of the following conditions and circumstances is MOST appropriate for the therapist to delegate to the assistant? 1. Ataxia, who is undergoing a trial to determine an appropriate assistive device 2. Hemiparesis, whose initial evaluation has not been completed 3. Multiple sclerosis, who is receiving gait training with a rolling walker 4. Alzheimer disease, who is easily agitated and is receiving initial gait training

3. Multiple sclerosis, who is receiving gait training with a rolling walker 1. A physical therapist would be required to make modifications to plans of care, such as determining an appropriate assistive device. 2. A physical therapist would be required to perform initial evaluations for patients. 3. Treatment of a patient who is stable and has an existing program would be most appropriate for delegation to the assistant. Therefore, treatment of the patient who has multiple sclerosis and who has an established gait training program would be the most appropriate to delegate. 4. It would be best for a physical therapist to treat a patient with a complicated condition receiving a new treatment, such as an easily agitated patient receiving initial gait training.

A physical therapist is examining a patient with right-sided heart disease who takes diuretics. Which of the following lower extremity conditions is MOST likely to be present? 1. Pallor 2. Shiny skin 3. Pitting edema 4. Absence of hair growth

3. Pitting edema 1. Pallor, or lack of color, is a characteristic of arterial disease, but is not associated with right-sided heart failure. 2. Shiny skin is a characteristic of arterial disease, but is not associated with right-sided heart failure. 3. When the right side of the heart starts to fail, fluid collects in the feet and lower legs, causing pitting edema. 4. Absence of hair growth is a characteristic of arterial disease, but is not associated with right-sided heart failure.

A patient with leukemia has developed thrombocytopenia after a bone-marrow transplant. Which of the following measures is indicative of the status of the thrombocytopenia? 1. T4 lymphocyte count 2. Red blood cell count 3. Platelet count 4. White blood cell count

3. Platelet count 1. The T4 lymphocyte count is used to assess immune status in patients who have human immunodeficiency virus or acquired immunodeficiency syndrome. 2. The red blood cell count is utilized to assess for the presence of anemia. 3. Thrombocytopenia is an acute or chronic decrease in the number of platelets in the circulation. 4. The white blood cell count would be examined to determine presence of infection or degree of immunosuppression.

A long-distance runner comes to physical therapy with insidious onset of lower leg pain. The patient's examination results reveal weakness of toe flexion and ankle inversion. The physical therapist suspects vascular compromise associated with this presentation. Palpation at which of the following locations is MOST likely to reveal diminished arterial pulses in the patient? 1. Dorsal aspect of the foot 2. Posterior to the lateral malleolus 3. Posterior to the medial malleolus 4. Lateral aspect of the popliteal fossa

3. Posterior to the medial malleolus 3. The patient has signs and symptoms consistent with posterior compartment syndrome. The posterior tibial artery is most likely to be involved in chronic posterior compartment syndrome. This artery should be palpated posterior to the medial malleolus.

Which of the following positions is BEST to assess the length of a patient's rectus femoris muscle? 1. Sidelying with tested hip in flexion 2. Supine with tested hip and knee in flexion 3. Prone with tested knee in flexion 4. Sidelying with tested hip in extension

3. Prone with tested knee in flexion 1. Hip flexion would shorten the rectus femoris. The position of the knee is important in assessing the length of the rectus femoris. 2. Hip flexion would shorten the rectus femoris. 3. Prone with the knee in flexion keeps the hip in neutral and does not allow the hip to flex. This position is used in the Ely test, which is used to assess rectus femoris tightness. 4. Sidelying with the tested hip in extension places the hip in the correct position, but knee position is also important in assessing the length of the rectus femoris.

As a patient progresses in learning a new motor skill, there should be an increasing reliance on which of the following types of feedback? 1. Tactile cueing 2. Visual knowledge of results 3. Proprioceptive input 4. Verbal information

3. Proprioceptive input 3. As the individual progresses from the Fitts and Posner cognitive stage of motor learning to the associative and autonomous stages, the learner increasingly relies on the ability to self detect and correct performance errors through intrinsic feedback (Umphred, p. 79). Proprioceptive input permits error corrections to occur during the movement (O'Sullivan, p. 398)

A physical therapist receives a telephone call from a woman who identifies herself as a friend of one of the therapist's patients. She wants to know how the patient is doing and whether the patient will be able to go up and down stairs, because she wants to take the patient home for a weekend visit. Which of the following actions is MOST appropriate for the therapist to take? 1. Discuss the patient's program and functional status with the caller. 2. Invite the caller to observe the patient's next therapy session. 3. Refuse to discuss the patient, unless the patient's permission is obtained. 4. Refer the caller to the patient's social worker.

3. Refuse to discuss the patient, unless the patient's permission is obtained. Information relating to the physical therapist/patient relationship is confidential. It may not be communicated to a third party not involved in the patient's care without the patient's prior written consent.

A patient with the diagnosis of astrocytoma malignancy is receiving physical therapy in the patient's home. While sitting in a chair at the dining room table, the patient begins to have a grand mal seizure. The physical therapist lowers the patient to the floor. Which of the following actions is the MOST appropriate NEXT step for the therapist? 1. Activate the emergency response system. 2. Protect the patient from biting the tongue by inserting a tongue blade. 3. Roll the patient to a sidelying position. 4. Extend the patient's neck for airway clearance.

3. Roll the patient to a sidelying position. 1. Activating the emergency response system is not the first step and may not be indicated until further assessment is completed after the seizure. 2. There is no need to try to keep the patient from biting the tongue or to insert an object into the mouth, which could cause damage to the patient's teeth. 3. During a seizure, injury should be prevented. Rolling the patient to sidelying position will help prevent aspiration. 4. Extending the patient's neck for airway clearance is not an appropriate response when a patient is having a seizure.

A patient who had a radial shaft fracture 8 weeks ago just had the cast removed. The patient has limited range of motion in all directions and limited strength of the wrist and finger joints. Which of the following interventions would be MOST appropriate to begin developing range and motor control of the patient's wrist and hand? 1. Closed-chain concentric wrist strengthening 2. Throwing and catching a ball with the affected hand 3. Tendon and nerve gliding exercises for the wrist and hand 4. Eccentric strengthening exercises of the wrist flexors and extensors

3. Tendon and nerve gliding exercises for the wrist and hand 1. Closed-chain concentric wrist strengthening would be introduced during a later phase of rehabilitation. A patient who has limited wrist and finger range of motion and strength would not be able to tolerate this activity. 2. Throwing and catching a ball with the affected hand would be introduced during a later phase of rehabilitation. A patient who has limited wrist and finger range of motion and strength would not be able to tolerate this activity. 3. Tendon and nerve gliding exercises are used to begin breaking up the adhesions that may have developed following a fracture. These exercises help to develop neuromuscular control and coordinate movement. 4. Eccentric strengthening exercises of the wrist flexors and extensors would be introduced during a later phase of rehabilitation. This activity would not be the most appropriate to use for range of motion and motor control directly after cast removal.

On the first day following a patient's total knee arthroplasty, a physical therapist begins treating the patient with a continuous passive motion device with a setting of 0° to 40° of motion. Which of the following reasons for using a continuous passive motion device is MOST appropriate for this patient? 1. To decrease length of the patient's hospital stay 2. To decrease incidence of deep vein thrombosis 3. To help the patient regain knee flexion 4. To prevent knee flexion contracture

3. To help the patient regain knee flexion 1. The evidence that a continuous passive motion device reduces hospital stay is variable. 2. There is no clear evidence that a continuous passive motion device prevents deep vein thrombosis. 3. Evidence suggests that a continuous passive motion device can help the patient to regain knee flexion. 4. If the knee is not placed in full extension outside the continuous passive motion device unit, a knee flexion contracture may result.

A physical therapist should anticipate that an abnormal lymph node will feel: 1. soft and nonmobile and have an increased skin temperature. 2. firm and nontender and have an increased skin temperature. 3. firm, mobile, and tender or nontender. 4. soft, mobile, and tender or nontender.

3. firm, mobile, and tender or nontender. 1. An abnormal lymph node would not feel soft, and an elevated skin temperature is not expected. 2. An abnormal lymph node may feel firm and nontender, but an elevated skin temperature is not an expected accompanying feature of an abnormal lymph node. 3. An abnormal lymph node can range in feeling from firm to hard, be mobile or nonmobile, and be tender or nontender. 4. An abnormal lymph node would not feel soft.

A patient with peripheral vascular disease comes to physical therapy for evaluation of leg pain that gets worse when walking. The patient will MOST likely also have: 1. relief of pain with the legs elevated. 2. purple or brown pigmentation of the skin on the legs. 3. relief of pain with the legs in the dependent position. 4. a positive Homans sign.

3. relief of pain with the legs in the dependent position. 1. The patient most likely has intermittent claudication caused by arterial insufficiency. Elevating the legs in the presence of arterial insufficiency decreases blood flow, which increases pain. 2. The patient most likely has intermittent claudication caused by arterial insufficiency. Purple or brown pigmentation of the skin on the legs is associated with venous insufficiency, not arterial insufficiency. 3. The patient most likely has intermittent claudication caused by arterial insufficiency. Placing the patient's legs in the dependent position facilitates blood flow and reduces pain. 4. Pain with exercise is indicative of intermittent claudication, not deep vein thrombosis, which is associated with a positive Homans sign.

A physical therapist is treating a patient with chronic range of motion limitation due to tight hamstrings. Which of the following applications of ultrasound and stretching is BEST to restore normal range of motion? 1. Pulsed ultrasound at 1 MHz with onset of stretching 10 minutes after the ultrasound treatment 2. Pulsed ultrasound at 1 MHz with stretching for 10 minutes during and immediately after the ultrasound treatment 3. Continuous ultrasound at 1 MHz with onset of stretching 10 minutes after the ultrasound treatment 4. Continuous ultrasound at 1 MHz with stretching for 10 minutes during and immediately after the ultrasound treatment

4. Continuous ultrasound at 1 MHz with stretching for 10 minutes during and immediately after the ultrasound treatment 4. Stretching should be done during and immediately after the ultrasound treatment. One MHz continuous ultrasound provides the greatest thermal effects and would facilitate the stretch.

A female patient had a radical mastectomy with axillary node removal. Which of the following signs would indicate initial development of lymphedema in the patient? 1. Increased blood pressure 2. Numbness in the antecubital fossa 3. Atrophy of the brachioradialis muscle 4. Decreased flexibility of the digits of the hand

4. Decreased flexibility of the digits of the hand 1. Changes in blood pressure are not an initial sign of lymphedema (Goodman, pp. 682-683). 2. Numbness of the affected body part is a possible symptom of lymphedema, but the antecubital fossa is not the most likely location (Goodman, p. 680; Drake, pp. 140-141) 3. Muscular atrophy, conceivably due to nerve compression from lymphedema, is an unlikely initial sign of lymphedema (Goodman, p. 680). 4. Decreased finger flexibility may be an initial sign of peripheral lymphedema

Excessive upward rotation of the right scapula is noted when a patient attempts to perform shoulder flexion. Which of the following exercises is MOST appropriate to help correct the excessive scapular rotation? 1. Right scapular protraction against resistance with the right arm at 90° of flexion 2. Bilateral scapular elevation with the upper extremities at 180° of flexion 3. Wall push-ups with an isometric hold at end range with the elbows extended 4. Bilateral scapular adduction with the upper extremities medially (internally) rotated and adducted across the back

4. Bilateral scapular adduction with the upper extremities medially (internally) rotated and adducted across the back 1. Right scapular protraction against resistance with the right arm at 90° of flexion would help strengthen the serratus anterior, an upward rotator of the scapula (p. 82). 2. Bilateral scapular elevation with the upper extremities at 180° of flexion would activate the upper trapezius as well as the rhomboids. Since the upper trapezius is an upward rotator of the scapula, this would not be the best exercise to use. (p. 88) 3. Wall push-ups with an isometric hold at end range with the elbows extended would help strengthen the serratus anterior, which would tend to aggravate the problem (p. 82). 4. Excessive upward rotation of the scapula can result from weakness of the rhomboids and latissimus dorsi (downward rotators). Scapular adduction with medial (internal) rotation and adduction of the arm would require action by those muscles. (p. 102)

A 39-year-old male patient with a 20-year history of smoking reports cramping pain of the buttock, thigh, and calf that is worse when walking. Which of the following findings is MOST likely present? 1. Forward stooping while walking 2. Pain in the calf with abrupt dorsiflexion 3. Brown pigmentation of the lower extremity 4. Blood pressure that is greater in the arm than in the leg

4. Blood pressure that is greater in the arm than in the leg 1. The patient is experiencing signs of intermittent claudication, which is associated with peripheral arterial disease. Forward stooping while walking is more indicative of a neurogenic disorder than intermittent claudication (p. 541). 2. The patient is experiencing signs of intermittent claudication, which is associated with peripheral arterial disease. Pain in the calf with abrupt dorsiflexion is a test that is could be used for a deep vein thrombosis (p. 650). 3. The patient is experiencing signs of intermittent claudication, which is associated with peripheral arterial disease. Hyperpigmented (bluish-brown) skin color often superior to the medial malleolus is a finding associated with a venous disorder (p. 656). 4. Peripheral arterial disease correlates most strongly with cigarette smoking. Male gender is also a risk factor. The patient is experiencing intermittent claudication pain, aching, or cramping of muscles, causing limping. Ischemia should be strongly suspected when systolic blood pressure is lower at the ankle than at the arm. (pp.

Evaluation of a 5-year-old child reveals difficulty in climbing stairs and rising from the floor, as well as proximal muscle weakness. Inspection shows a protuberant abdomen, increased lumbar lordosis, and pseudohypertrophy of both calves. These clinical findings are consistent with a diagnosis of: 1. Legg-Calvé-Perthes disease. 2. hypotonic cerebral palsy. 3. Charcot-Marie-Tooth disease. 4. Duchenne muscular dystrophy.

4. Duchenne muscular dystrophy. 4. Duchenne muscular dystrophy is characterized by enlargement of posterior calves, weakness of proximal muscle groups, and difficulty with stair climbing and rising from the floor (Palisano, pp. 245-246).

Which of the following recommendations is MOST appropriate for a patient who has a diagnosis of diabetes mellitus and sensory neuropathies in both feet? 1. Perform weekly foot examinations to check for possible skin breakdown. 2. Frequently wash and apply lotion to the entire foot and wear soft slippers. 3. Avoid wearing soft socks with shoes. 4. Ensure comfortable and proper shoe fit and break in new shoes slowly.

4. Ensure comfortable and proper shoe fit and break in new shoes slowly. 1. Patients who have diabetic neuropathies should examine their feet daily. 2. The diabetic foot should have adequate support, and lotion should be avoided between the toes. 3. The feet should always be protected by soft socks and supportive shoes. 4. Undue friction or pressure on a foot that has sensory deficits is a cause of ulceration.

A physical therapist is setting up a home program of electrical stimulation for a patient who has Bell palsy. Which of the following muscles should be stimulated as part of the home program? 1. Sternocleidomastoid 2. Masseter 3. Temporalis 4. Frontalis

4. Frontalis 1. The sternocleidomastoid is innervated by the spinal accessory nerve (CN XI). 2. The masseter is innervated by the trigeminal nerve (CN V). 3. The temporalis is innervated by the trigeminal nerve (CN V). 4. Bell palsy involves the facial nerve (CN VII). The frontalis is the only muscle listed that is innervated by the facial nerve (CN VII).

A patient exhibits weakness throughout the right lower extremity with normal strength on the left side. Sensory testing shows a loss of pressure sensation over the right thigh and leg and a loss of pain and temperature sensation over the left thigh and leg. The patient has a positive Babinski sign on the right. Which of the following associated findings is MOST likely to be found during further examination of this patient? 1. Presence of clonus in the left ankle 2. Marked atrophy of the right lower extremity muscles 3. Spasticity in the left lower extremity 4. Increased deep tendon reflexes on the right side

4. Increased deep tendon reflexes on the right side 4. The patient's signs and symptoms are consistent with hemisection of the spinal cord, or Brown-Séquard syndrome. The lesion is on the right side of the spinal cord, and it is an upper motor neuron lesion, indicated by the positive Babinski sign, with damage to the corticospinal tract, posterior columns, and lateral spinothalamic tract. Upper motor neuron spinal cord lesions present with hyperactive tendon reflexes (in this case on the right side), clonus, and spasticity ipsilateral to the side of the lesion. Pressure sensation is lost on the ipsilateral side, and pain and temperature sensation are lost contralateral to the side of the lesion due to the crossing of those fibers in the spinal cord. (Schenkman, pp. 231, 275)

A 6-month-old infant who has Erb palsy is MOST likely to demonstrate which of the following impairments? 1. Flaccid bilateral upper extremities 2. Involved forearm postured in supination 3. Elbow flexion contracture of the involved arm 4. Involved arm postured in shoulder medial (internal) rotation

4. Involved arm postured in shoulder medial (internal) rotation 1. Erb palsy involves injury to the upper roots of the brachial plexus at C5 and C6 and results in unilateral paralysis, not bilateral paralysis. 2. Erb palsy involves injury to the upper roots of the brachial plexus at C5 and C6. Erb palsy results in positioning the affected limb in pronation, not supination. 3. Erb palsy involves injury to the upper roots of the brachial plexus at C5 and C6. It is unilateral and involves paralysis, not spasticity that would result in a contracture. 4. Erb palsy is characterized by shoulder weakness resulting in the involved arm postured in medial (internal) rotation of the shoulder.

A physical therapist is planning to use functional electrical stimulation to decrease shoulder subluxation in a patient post stroke. Which of the following techniques is MOST appropriate? 1. Use direct current. 2. Limit on times to less than 10 seconds. 3. Adjust electrode placement to achieve upper trapezius muscle contraction. 4. Place electrodes over the posterior deltoid and supraspinatus muscles.

4. Place electrodes over the posterior deltoid and supraspinatus muscles. 1. Direct current is not well suited for this functional electrical stimulation. A pulsed current (symmetrical or asymmetrical biphasic pulsed current) is recommended for treatment of shoulder subluxation. 2. On times up to 30 seconds can be used if tolerated by muscles or if pulsed current is used. The goal is to increase on time and decrease off time as muscle endurance improves. 3. Activation of the trapezius muscle should be avoided in a patient who has shoulder subluxation as it will elevate the scapula and not strengthen the glenohumeral muscles. 4. Recommended electrode placement for reducing shoulder subluxation is over the posterior deltoid and supraspinatus muscles.

A patient with Parkinson disease demonstrates shortness of breath with activity. Which of the following tests is MOST appropriate for a physical therapist to perform to help delineate a cause for the patient's shortness of breath? 1. Deep tendon reflex testing 2. Sensory examination 3. Muscle strength testing 4. Posture examination

4. Posture examination 4. With regard to posture examination, as bradykinesia and rigidity evolve in Parkinson disease, concomitant kyphosis also develops. Thoracic kyphosis contributes to a restriction in ventilation and subsequent shortness of breath.

Which of the following elements of motor learning contributes MOST to retention of a motor skill? 1. Mental practice 2. Manual contact to guide the patient 3. Summary knowledge of results 4. Practice of the motor skil

4. Practice of the motor skill 1. Mental practice can help in the performance of a task, but physical practice is best. 2. Manual contact is good for initial learning of a motor task but is not associated with long-term retention. 3. Summary knowledge of results is better for application of a skill under other conditions. 4. Generally, the more practice a patient has, the more the patient learns. Therefore, for optimal retention, ample practice sessions should be available while avoiding fatigue.

A patient with a severe cerebellar lesion can sit independently, can stand with minimal assistance, and requires moderate assistance of one person to walk safely. The patient wants to transfer independently from a wheelchair to a bed. Which of the following transfer techniques is MOST appropriate for the patient? 1. Standing pivot 2. Manual pivot 3. Mechanical lift 4. Sliding

4. Sliding 1. Standing-pivot transfer requires standing, which the patient is not able to safely perform alone. 2. Manual-pivot transfer is not appropriate for a patient who is not dependent. 3. Mechanical-lift transfer is not appropriate for a patient who is not dependent. 4. Sliding transfer is the safest independent technique because the patient cannot stand without minimal assistance.

A patient is receiving physical therapy intervention for rheumatoid arthritis, which is in remission. Which of the following interventions is MOST appropriate for the patient? 1. Contract-relax stretching for tight structures 2. End-range mobilization techniques 3. Elimination of functional activities of involved parts 4. Strengthening exercises for weak muscles

4. Strengthening exercises for weak muscles 1. Contract-relax stretching for tight structures is not recommended for soft tissues compromised by rheumatoid arthritis. 2. End-range mobilization techniques are contraindicated for soft tissues compromised by rheumatoid arthritis. 3. The elimination of functional activities is not encouraged. 4. Strengthening exercises are the most appropriate intervention for a patient who has rheumatoid arthritis in remission.

A physical therapist is examining hip range of motion in a patient as shown in the photograph. Passive range of motion is applied to the patient's legs in the direction of the arrows. The photograph shows the end points of the range of motion. The MOST likely cause of the hip dysfunction occurring in the patient is: 1. hypomobility of the hip medial (internal) rotators on the left. 2. weakness of the left hip medial (internal) rotators. 3. laxity of the left hip capsule. 4. tightness of the hip lateral (external) rotators on the left

4. tightness of the hip lateral (external) rotators on the left 4. tightness of the hip lateral (external) rotators on the left Medial (internal) rotation of the left hip is limited. This could be caused by hypomobility of the left hip capsule or tightness of the left lateral (external) rotators, especially the piriformis.

A physical therapist is treating a patient who had knee surgery after a motor vehicle accident. The patient is suing the driver of the other vehicle and has hired an attorney. The patient is being treated in physical therapy as an outpatient 3 times/week. After the third session, a relative of the patient calls and wants to know how much weight the patient can bear on the leg. The therapist should: 1. seek clearance from the attorney before informing the patient's relative. 2. describe the weight-bearing limits in general terms. 3. have the patient's relative call the medical records office. 4. ask the patient's relative to visit directly with the patient.

4. ask the patient's relative to visit directly with the patient. 1. Authorization from the patient, not the attorney, is required to release information regarding the patient's status. 2. Giving the caller information about the patient's case is a violation of the patient's privacy. 3. The physical therapist would be more appropriate as the source of this information than the medical records office. The therapist cannot provide this information without the patient's expressed or written permission. 4. A patient must give informed consent regarding information about the treatment. The best alternative for the therapist is to have the patient's relative visit directly with the patient or have the patient's relative attend a treatment session with the patient's permission. Informed consent can only be given by the patient, unless the patient is incapable of doing so. In this case, there is nothing to suggest that the patient is unable to give consent.

A patient who has a chronic L5-S1 disc protrusion reports low back pain and weakness of the right lower extremity. Results of which of the following examination components would MOST likely be abnormal? 1. Achilles reflex 2. Babinski reflex 3. Hamstring flexibility 4. Ankle dorsiflexion strength

1. Achilles reflex 1. An L5-S1 disc protrusion would likely cause S1 nerve root compression and S1 reflex changes. The Achilles reflex is an S1 reflex. (p. 1466) 2. An abnormal Babinski reflex would indicate upper neuron involvement (p. 134). 3. Hamstring flexibility is not directly related to an L5-S1 disc protrusion (p. 1494). 4. Dorsiflexion strength would be impaired with an L4 nerve root condition (p. 1494).

Which of the following descriptions BEST characterizes stable angina? 1. Episodes of nonradiating chest pain each lasting 5-15 minutes 2. Episodes of severe chest pain each lasting longer than 15 minutes 3. Chest pain occurring at rest and unaffected by exertion 4. Chest pain accompanied by dysrhythmias

1. Episodes of nonradiating chest pain each lasting 5-15 minutes 1. Stable angina generally occurs during physical effort and is characterized by substernal, usually nonradiating pain lasting between 5 and 15 minutes. 2. In unstable angina, the episodes occur during physical exertion or psychological stress and are more frequent, the pain may be severe, and the duration of each event is usually greater than 15 minutes. 3. Variant angina occurs while the individual is at rest, usually during waking and at the same hour of the day. 4. Dysrhythmias occur more commonly in individuals who have variant angina than in those with exertional angina (either stable or unstable).

During a posture examination, the physical therapist notes that both of the patient's patellae point inward when viewed from the front of the patient. The MOST likely cause of this problem is excessive: 1. femoral anteversion. 2. weakness of the vastus medialis. 3. genu varum. 4. medial tibial torsion.

1. femoral anteversion. 1. The most common cause of inwardly pointing or "squinting patellae" is excessive femoral anteversion. Although there is normally 8° to 15° of femoral anteversion, an excessive amount leads to squinting patellae and toeing-in. The other options would all have a tendency to cause the patellae to point outward during standing. 2. Weakness of the vastus medialis would be more likely to cause the patellae to point outward during standing. 3. Genu varum would be more likely to cause the patellae to point outward during standing. 4. Medial tibial torsion would be more likely to cause the patellae to point outward during standing

In splinting or immobilization, the functional position of the hand includes wrist extension, phalangeal: 1. flexion, and abduction of the thumb (1st digit). 2. extension, and abduction of the thumb (1st digit). 3. extension, and adduction of the thumb (1st digit). 4. flexion, and adduction of the thumb (1st digit).

1. flexion, and abduction of the thumb (1st digit). The functional position of the wrist and hand describes the position from which the optimal function is most likely to occur. This position is described as: (1) slight wrist extension, (2) slight ulnar deviation, (3) fingers flexed at the MCP, PIP, and DIP joints, and (4) thumb (1st digit) slightly abducted.

The PRIMARY purpose of pursed-lip breathing is to: 1. help prevent the collapse of pulmonary airways during exhalation. 2. decrease the removal of carbon dioxide during ventilation. 3. increase the residual volume of respiration so that more oxygen is available for body metabolism. 4. stimulate further mobilization of mucous secretions to higher air passages where they can be expectorated.

1. help prevent the collapse of pulmonary airways during exhalation. 1. A patient who has chronic obstructive pulmonary disease has premature collapse of the airways upon exhalation, which leads to air trapping and ultimately poor gas exchange. Breathing out through pursed lips slows the airflow and creates a back pressure, which helps to prevent the airways from collapsing while exhaling. (pp. 115, 550)

When held supported in standing position, a 14-month-old child with spastic diplegia is up on tiptoes with the toes curled. This position is characteristic of a: 1. proprioceptive placing reaction. 2. Moro reflex. 3. plantar grasp reflex. 4. traction response.

3. plantar grasp reflex. 1. A proprioceptive placing reaction would be exhibited by the lower extremity when the shin is rubbed against a surface resulting in the flexion of the hip and knee, not the toes curling (Ropper, p. 592). 2. The Moro reflex is exhibited by upper extremity movement in response to passive positioning of the neck (Effgen, p. 70). 3. The plantar grasp reflex is characterized by curling of the toes when a child is held supported in standing position. The reflex is normal up to 9 months of age. Delayed integration of this reflex can result in delayed independent ambulation. (Effgen, p. 70) 4. Traction response is exhibited when traction applied to the arm results in upper extremity flexion (Cikulin-Kulinski, p. 483).

A patient who is relearning the task of moving from sitting to standing position following traumatic brain injury is frustrated because of repeated failed attempts. To facilitate the patient's success, a physical therapist should FIRST: 1. permit the patient to rest until the next physical therapy session and reattempt the activity. 2. encourage the patient to visualize success with the task before resuming attempts. 3. provide incentive by holding a desired object for the patient to reach toward. 4. decrease the challenge of the task, so that the patient experiences success.

4. decrease the challenge of the task, so that the patient experiences success. 1. Stopping the session upon failure may further frustrate the patient. 2. Visualization, although useful, is a higher level task that should not be the first strategy used. 3. Poor body mechanics and stimulation of tone may occur if the patient reaches forward while moving from sitting to standing position. 4. It is most important for the patient to experience some form of success in order to provide motivation. Necessary to learning are motivation to try the unknown and, simultaneously, success in learning, to retain the learner's motivation.

A physical therapist is planning a patient education program for a group of patients with chronic low back pain. The therapist should use which of the following strategies to MOST increase the likelihood that the patients will utilize the proper body mechanics they learn? 1. Ask the patients to demonstrate use of proper body mechanics. 2. Provide a reference list of articles describing body mechanics. 3. Ask the patients to describe actions they do that increase their back pain. 4. Provide information on the frequency of low back injuries due to improper body mechanics.

1. Ask the patients to demonstrate use of proper body mechanics. 1. Evaluation of the patients' retention of the information presented in the program can be enhanced by asking the patients questions about the program information, having the patients ask questions about the program, having the patients demonstrate what they have learned, and testing the patients about the program material (p. 35). 2. Providing a list of articles does not require active participation and does not ensure recall or understanding by the patient. 3. Asking the patient about causes of the pain is important in the patient assessment but does not include any education about ways to reduce pain by using better body mechanics. This is not active learning. 4. Education about the frequency of back injuries does not include any active learning for the patient and will not improve the patient's body mechanics without demonstration and recall of proper technique.

Increased residual volume is LEAST likely to be a finding in pulmonary function testing of a patient with which of the following conditions? 1. Atelectasis 2. Bronchiectasis 3. Chronic bronchitis 4. Emphysema

1. Atelectasis 1. Atelectasis is more of a restrictive issue and would not lead to an increase in residual volume. Restrictive lung disease is associated with a decreased residual volume. 2. Bronchiectasis is a condition that leads to obstructive problems. Obstructive lung disease is associated with an increased residual volume. 3. Chronic bronchitis is a condition that leads to obstructive problems. Obstructive lung disease is associated with an increased residual volume. 4. Emphysema is a condition that leads to obstructive problems. Obstructive lung disease is associated with an increased residual volume.

During an examination of elbow strength using manual muscle testing, a patient supinates the forearm when attempting elbow flexion. Which of the following muscles is MOST likely doing the major part of the work? 1. Biceps brachii 2. Brachialis 3. Supinator 4. Brachioradialis

1. Biceps brachii 1. The biceps brachii is both an elbow flexor and supinator, and it is most effective as a supinator with the elbow flexed to about 90° (approximately the muscle testing position). Therefore, when the elbow both flexes and supinates, the biceps brachii would be the most likely muscle causing this action. (p. 715) -The supinator does not flex the elbow

During walking, a patient demonstrates backward leaning of the trunk at right heel strike (initial contact). The result of manual muscle testing of the weak muscle is Good minus (4-/5). Which of the following exercises BEST addresses this patient's muscular weakness? 1. Bridging activities progressing from double leg to single leg 2. Single-leg stance allowing the opposite side of the pelvis to drop, then return to a level position 3. Prone leg lifts with the leg straight 4. Standing on the uninvolved leg to perform active hip extension of the involved leg

1. Bridging activities progressing from double leg to single leg 1. A posterior trunk lean is associated with weak hip extensors (especially the gluteus maximus). Double and single leg bridging would specifically target the hip extensors and gluteus maximus. 2. Single-limb stance with pelvic drop and hike would focus on strengthening the gluteus medius muscle, a hip abductor. 3. Prone leg lifts would not provide enough resistance to cause physical adaptation of the hip extensors. 4. Standing hip extension would not provide enough resistance to cause physical adaptation.

A physical therapist plans to use a tilt table for a patient who is having difficulty tolerating upright sitting position. The therapist should stop inclining the tilt table if the patient experiences which of the following signs and symptoms? 1. Decrease in diastolic blood pressure of 15 mm Hg 2. Increase in systolic blood pressure of 10 mm Hg 3. Increase in heart rate of 15 bpm 4. Decrease in oxygen saturation to 93

1. Decrease in diastolic blood pressure of 15 mm Hg 1. Excessive drop in blood pressure is indicative of patient intolerance to upright posture. 2. Slight increases in systolic blood pressure do not indicate intolerance of upright position. 3. Heart rate increases are normal and do not indicate intolerance of upright position. 4. Oxygen saturation of 93% is within the acceptable range.

What is the close-packed position of the hip? 1. Medial (internal) rotation with extension and abduction 2. Extension with adduction and lateral (external) rotation 3. Lateral (external) rotation with flexion and abduction 4. Flexion with adduction and lateral (external) rotation

1. Medial (internal) rotation with extension and abduction The close-packed position for the hip is medial (internal) rotation with extension and abduction. An alternative position is 90° of hip flexion with abduction and slight lateral (external) rotation.

A patient who has low back pain reports a history of gastroesophageal reflux disease. Which of the following exercise modifications should be used to prevent exacerbations of the reflux disease? 1. Position the patient in an upright posture. 2. Place the patient in prone position. 3. Position the patient on the right side instead of the left. 4. Place the patient in supine position.

1. Position the patient in an upright posture. 1. Individuals who have gastroesophageal reflux disease usually have more problems in the supine position. Positioning during exercise toward the upright position should be helpful. 2. The prone position increases abdominal pressure, which exacerbates gastroesophageal reflux disease. 3. Individuals who have gastroesophageal reflux disease usually have more problems when lying on the right side. This tends to straighten the esophagus, allowing for more reflux to occur. 4. The supine position should be avoided, particularly after eating.

A patient who reports double vision has ptosis, lateral strabismus, and a dilated pupil in the left eye. Which of the following cranial nerve test results is MOST likely to be abnormal? 1. Pupillary light reflex 2. Facial muscle strength 3. Jaw-jerk reflex 4. Pain sensation in the face

1. Pupillary light reflex the cranial nerve involved is the oculomotor nerve (CN III). This nerve innervates the medial rectus, which, if weak, would cause a lateral strabismus. The oculomotor nerve (CN III) is also responsible for mediating pupillary constriction, and a lesion would cause pupillary dilation. The ptosis is caused by loss of innervation to the levator palpebrae superioris muscle, which elevates the eyelid. The double vision would be caused by the inability to move the eyeball normally, because four of the six ocular muscles are controlled by the oculomotor nerve (CN III). The oculomotor nerve (CN III) is also important in mediating the pupillary light reflex.

A patient spilled boiling water on the right arm when reaching for a pan on the stove. The forearm, elbow, and lower half of the upper arm appear blistered and red, with some subcutaneous swelling, and are painful when touched. To facilitate optimal function, which of the following interventions should a physical therapist PRIMARILY emphasize? 1. Range of motion exercises to the right hand, wrist, and elbow 2. Application of compression dressings 3. Sterile whirlpool to the right upper extremity 4. Splinting of the right upper extremity in 10° of elbow flexion and forearm pronation

1. Range of motion exercises to the right hand, wrist, and elbow 1. The patient will be reluctant to move the limb because it is painful. Range of motion exercises are essential to regain and maintain a functional range of motion. In this case, the physical therapist's primary goal is to restore function with range of motion exercises. 2. Application of compression dressings would be primarily used for reducing edema but would not be directly related to restoring function. 3. Whirlpool therapy would not promote function as much as range of motion exercises. In addition, whirlpool is considered a form of mechanical debridement and is not indicated for wounds without necrotic tissue. 4. If splinting were needed, the preferred position would be full elbow extension with forearm supination.

A patient with a recent ankle injury reports anterior ankle pain while walking. Pain is reproduced ONLY during passive rotation of the lower leg. The patient most likely sustained which of the following injuries? 1. Syndesmosis sprain 2. Calcaneocuboid sprain 3. Fibular shaft fracture 4. Tibial stress fracture

1. Syndesmosis sprain 1. Passive rotation of the lower leg tests the integrity of the tibiofibular syndesmosis. With a positive test result, the patient will have reproduction of symptoms and, in this case, anterior ankle pain. 2. The calcaneocuboid joint could be compressed with lateral (external) rotation of the tibia, but this would not stress the ligaments. 3. A patient who has a fibular shaft fracture would have tenderness over the fracture site and pain with weight-bearing during the initial stage of stance. 4. Passive rotation of the lower leg does not load the tibia and/or muscles enough to provoke pain from a stress fracture.

In a research study, a correlation coefficient of 0.30 was found for the relationship between two variables. Which of the following interpretations of this finding is MOST appropriate? 1. The variables have a low correlation. 2. Thirty percent of the variability in one variable can be accounted for by the other variable. 3. There are no significant differences between the variables. 4. There is low positive predictive value between the variables.

1. The variables have a low correlation. 1. A correlation coefficient of 0.26 to 0.49 is considered low (p. 321). 2. Correlation is not causation. When a high correlation exists, an individual's score on one variable is related to the score of the other variable. A correlation coefficient of 0.30 indicates a low correlation. (pp. 318-319) 3. Correlation only determines if there is a relationship between two variables. Significant differences are calculated using p values. (p. 272) 4. Correlation is not used in determining positive predictive values. Positive predictive value is the likelihood that a condition is present if a test has a positive result. (pp. 187-188)

Treatment of a patient with hemophilia who has a subacute hemarthrosis of the knee should INITIALLY include: 1. active assistive range-of-motion exercise to the knee. 2. instruction of the patient for weight-bearing to tolerance. 3. gentle resistive range-of-motion exercise to the knee. 4. continuous immobilization of the knee in an extension splint.

1. active assistive range-of-motion exercise to the knee. 1. In this stage of hemarthrosis, there is still some bleeding into the joint space, but it is not as extensive as during the acute phase. Therefore, the patient will benefit from range-of-motion exercise to prevent contracture. The patient may need active-assistive exercises, because there may still be pain or edema in the joint that prevents independent performance of range of motion. 2. The mechanical trauma of weight-bearing to tolerance at this stage may impinge on and damage the pathologic synovium within the joint. 3. Resistive range of motion is more appropriate when pain and swelling have subsided and bleeding is not occurring. 4. Continuous immobilization in the extended position will promote contracture in the edematous knee

A 3-month-old infant has poor midline head control. During evaluation, the physical therapist notes facial asymmetry and observes that the infant holds the head in cervical rotation to the left and cervical lateral flexion to the right. The infant MOST likely has: 1. right congenital muscular torticollis. 2. left congenital muscular torticollis. 3. right cervical facet hypomobility. 4. left cervical facet hypomobility.

1. right congenital muscular torticollis. 1. The infant exhibits signs of torticollis affecting the right sternocleidomastoid muscle. Torticollis is named for the side of the affected sternocleidomastoid. 2. The infant exhibits signs of right sternocleidomastoid tightness, which would be described as right torticollis. 3. Asymmetry is not typically seen with cervical facet hypomobility in infants. 4. Asymmetry is not typically seen with cervical facet hypomobility in infants.

A postural correction program for a patient with forward head, kyphosis, and increased lumbar lordosis should include all of the following EXCEPT: 1. strengthening the scapular protractors. 2. strengthening the thoracic erector spinae muscles. 3. lengthening the short suboccipital muscles. 4. lengthening the lumbar erector spinae muscles.

1. strengthening the scapular protractors. 1. With this particular posture, the patient's scapulae would be in a protracted (abducted) position; therefore, the scapular protractors are already overactive and would require stretching, not strengthening. 2. The kyphosis suggests that the thoracic erector spinae muscles are weak and need strengthening. 3. The forward head posture suggests that the cervical spine is flexed and the occiput is extended; therefore, stretching of the suboccipital muscles would be indicated. 4. The lumbar lordosis indicates shortened lumbar erector spinae muscles; therefore, stretching of the lumbar erector spinae muscles would be indicated.

A physical therapist is monitoring the exercise of a patient who has type 1 diabetes. The patient's blood glucose level would be BEST for safe exercise at which of the following values? 1. 60 mg/dL (3.3 mmol/L) 2. 175 mg/dL (9.7 mmol/L) 3. 260 mg/dL (14.4 mmol/L) 4. 345 mg/dL (19.1 mmol/L)

2. 175 mg/dL (9.7 mmol/L) 1. A blood glucose level of 60 mg/dL (3.3 mmol/L) is considered hypoglycemic and is not safe for exercise. 2. Patients who have type 1 diabetes use insulin. Blood glucose levels should be between 100 and 250 mg/dL for patient safety. 3. A blood glucose level of 260 mg/dL (14.4 mmol/L) is hyperglycemic and is not safe for exercise. 4. A blood glucose level of 345 mg/dL (19.1 mmol/L) is hyperglycemic and is not safe for exercise.

Which of the following subjective reports from a patient with rheumatoid arthritis indicates the need for further medical examination by a physician? 1. Morning pain in both lower extremities 2. Numbness in both lower extremities 3. Increased pain during activities of daily living 4. Warmth and tenderness in the hand joints

2. Numbness in both lower extremities 1. Morning pain is a typical finding in rheumatoid arthritis. 2. Symmetrical numbness could be indicative of myelopathy from either infectious or neoplastic causes. A patient who has this symptom would require examination by a physician. 3. Increased pain during activities of daily living is also a typical finding in rheumatoid arthritis. 4. Tender, warm, swollen joints are typical features of an inflammatory arthritis. If the joint was hot and swollen, it would be suggestive of an infection and would require further medical evaluation.

An older adult patient had a total hip arthroplasty following a hip fracture from a fall in his home. The patient lived alone and was previously able to perform all activities of daily living independently. He has been admitted to a skilled nursing facility for rehabilitation. The patient's goal is to return home. His family believes he should be admitted to a long-term care facility for his own safety after acute rehabilitation is completed. Which of the following actions would be MOST appropriate to initiate FIRST? 1. Advise the family that the patient should determine his own discharge environment. 2. Recommend a team conference with the patient and his family to discuss discharge plans. 3. Schedule a home visit to determine if home modifications are needed before discharge. 4. Implement a treatment plan with a long-term goal of discharge to home. An older adult patient had a total hip arthroplasty following a hip fracture from a fall in his home. The patient lived alone and was previously able to perform all activities of daily living independently. He has been admitted to a skilled nursing facility for rehabilitation. The patient's goal is to return home. His family believes he should be admitted to a long-term care facility for his own safety after acute rehabilitation is completed. Which of the following actions would be MOST appropriate to initiate FIRST? 1. Advise the family that the patient should determine his own discharge environment. 2. Recommend a team conference with the patient and his family to discuss discharge plans. 3. Schedule a home visit to determine if home modifications are needed before discharge. 4. Implement a treatment plan with a long-term goal of discharge to home.

2. Recommend a team conference with the patient and his family to discuss discharge plans. 1. Allowing the patient to determine his own discharge environment may be fine if there are no safety concerns; however identifying any safety concerns the family has prior to discharge would be more appropriate. 2. The best answer would be for the therapist to recommend a team conference that would bring together members of the health care team, as well as the patient and family members, to discuss the patient's status and any safety concerns the family may have. 3. Scheduling a home visit would be more effective if the family's concerns have been identified prior to the home visit. 4. The implementation of a treatment plan for home care would be more effective if the family's concerns have been identified prior to the home visit.

A patient who has an Osgood-Schlatter lesion is MOST likely to experience pain around which of the following structures and with which activity? 1. Patella after jumping 2. Tibial tubercle after jumping 3. Patella after extended sitting 4. Tibial tubercle after extended sitting

2. Tibial tubercle after jumping 1. Pain at the patellar tendon after jumping would be characterized as jumpers knee or patellar tendinopathy. Although in the same region, the pain would not be characterized at the tibial tubercle as in an Osgood-Schlatter lesion. (Brukner, pp. 700-701) 2. Osgood-Schlatter lesion is an osteochondritis that occurs at the growth plate of the tibial tuberosity. It is extremely common in adolescents at the time of the growth spurt. It is usually associated with repeated knee extension in running and jumping and with pain around the tibial tuberosity, and it is aggravated by exercise. (Brukner, p. 896) 3. Pain after extended sitting is a symptom of patellofemoral syndrome due to increased patellofemoral compression when sitting (Brukner, p. 685). Pain is not at the tibial tubercle with a patellofemoral disorder as it is in an Osgood-Schlatter lesion. 4. In patients who have an Osgood-Schlatter lesion, pain around the tibial tuberosity is aggravated by exercise (Brukner, p. 897). Pain that is worsened by extended sitting is more characteristic of patellofemoral pain syndrome (Dutton, pp. 1039-1040).

A physical therapist examines a patient with multiple sclerosis who is in a period of exacerbation. The patient is independent with bed mobility, can sit unassisted at the edge of the bed, and requires physical assistance to stand with a walker. Which of the following interventions would have the HIGHEST priority? 1. Wheelchair propulsion up a 10-ft (3-m) ramp 2. Wheelchair transfers 3. Walking with an assistive device 4. Tub transfers

2. Wheelchair transfers 1. Wheelchair transfers are the means to enable the patient to be independently mobile and will be a requirement before more difficult tasks are performed, such as ascending a 10-foot (3-m) ramp. 2. Physical therapy intervention should focus on helping the patient obtain maximal functional independence. Wheelchair transfers are the means to enable the patient to be independently mobile and will be a requirement before more difficult tasks are performed. 3. The patient is presumably too weak to walk at this point in time. 4. Tub transfers are important, but the wheelchair will be the best means to get the patient to the tub. Therefore, for the initial intervention session, wheelchair transfers would be the most important.

A patient is doing active and resistive exercises on a mat table in the physical therapy department. After 15 minutes, the patient becomes short of breath, begins coughing, and expectorates pink, frothy sputum. At this point, the physical therapist should first stop the treatment, then NEXT: 1. assess vital signs, let the patient rest a few minutes with the feet elevated, and then resume with a less vigorous program. 2. sit the patient up, assess vital signs, and call a nurse or physician for further instructions. 3. lay the patient supine, transfer the patient to a stretcher, and return the patient to the nursing unit. 4. lay the patient down flat, call for assistance, and begin cardiopulmonary resuscitation

2. sit the patient up, assess vital signs, and call a nurse or physician for further instructions. 2. The presence of dyspnea and the pink, frothy sputum would suggest the presence of heart failure and resultant pulmonary edema. Heart failure can occur from poor cardiac muscle function as a result of myocardial infarction. Pulmonary edema occurs from the backflow of blood from the heart into the pulmonary vessels, increasing pulmonary capillary pressure. The increase in pulmonary capillary pressure increases fluid movement into the alveoli, which are normally dry. This leads to the presence of pink, frothy sputum that can be expectorated, along with shortness of breath (dyspnea). Positions that increase blood flow to the heart, such as lying flat, will increase the signs and symptoms. Therefore, the patient should be positioned with the head up or should be placed in a sitting position to help alleviate the symptoms.

A physical therapist is reviewing the medical record of a patient in the intensive care unit. The patient was admitted the previous night through the emergency department after a motorcycle accident resulting in a fractured right femur. The therapist notes a physician's order for a Doppler study of the left leg. The therapist should: 1. proceed with the evaluation and intervention without any restrictions. 2. withhold physical therapy until results of the study are obtained and interpreted by the physician. 3. proceed with the evaluation and limit intervention to transfer to a bedside chair. 4. obtain clearance from the nurse to provide intervention for the patient.

2. withhold physical therapy until results of the study are obtained and interpreted by the physician. 1. A physician's order for a Doppler study indicates possible deep vein thrombosis. A complete physical therapy evaluation and treatment is contraindicated due to a possible deep vein thrombosis. 2. A physician's order for a Doppler study indicates possible deep vein thrombosis. Physical therapy should not be conducted until the Doppler study is completed and the results analyzed by the physician. 3. A physician's order for a Doppler study indicates possible deep vein thrombosis. Transfer from bed to chair is contraindicated due to possible deep vein thrombosis. 4. The nurse alone should not be providing clearance; the Doppler study must be completed and the results interpreted by the physician.

A patient is asked to grip a card between the thumb (1st digit) and index finger (2nd digit) with both hands. The physical therapist pulls on the card in the direction of the arrow shown in the photograph. As the therapist pulls on the card the patient's right thumb (1st digit) flexes at the interphalangeal joint. The results indicate weakness in which of the following muscles? 1. Flexor pollicis longus 2. Abductor pollicis brevis 3. Adductor pollicis 4. Extensor pollicis longus

3. Adductor pollicis The test shown in the photograph is the Froment test. Both thumbs (1st digits) should stay extended during the test. If the thumb (1st digit) flexes, it is indicative of weakness of the adductor pollicis with substitution by the flexor pollicis longus, which is usually due to a lesion of the ulnar nerve.

During evaluation of a patient's balance, a physical therapist gently pushes the patient backward slightly and observes how the patient recovers from the perturbation. What strategy is the patient MOST likely to use to correct for the perturbation? 1. Knee 2. Hip 3. Ankle 4. Stepping

3. Ankle 1. For larger perturbations, individuals utilize hip and knee muscles to recover balance. 2. For larger perturbations, individuals utilize hip and knee muscles to recover balance. 3. For slight perturbations, most individuals use an ankle strategy. Ankle musculature is used to control the perturbation and recover balance. 4. If the perturbation is strong enough to cause the individual's center of mass to move outside the base of support, a stepping strategy would be employed by taking a step and increasing the size of the base of support.

A physical therapist is measuring the shoulder passive joint range of motion of a patient who has acute bursitis. What type of joint end-feel is MOST likely to occur? 1. Firm 2. Hard 3. Empty 4. Boggy

3. Empty 1. Firm end-feels would be expected due to increased muscular tone or capsular/muscular/ligamentous shortening. 2. Hard end-feels are expected due to chondromalacia, osteoarthritis, loose bodies in the joint, myositis ossificans, or displaced fracture. 3. Acute pain in a joint, as seen in bursitis, results in an abnormal end-feel. An empty end-feel is noted when the patient does not allow the therapist to take the joint through the full range of motion due to pain, muscle guarding, or muscle spasm. Examples of situations leading to this type of end-feel include acute joint inflammation, bursitis, abscess, fracture, or a psychogenic disorder. 4. Boggy end-feels would be expected due to fluid within joints, such as blood

A patient experiencing a recent onset of upper extremity lymphedema is MOST likely to have which of the following findings? 1. Pallor 2. Shiny skin 3. Pitting edema of the dorsal hand 4. Nonpitting fibrotic edema in the forearm and hand

3. Pitting edema of the dorsal hand 1. Pallor is associated with conditions resulting in arteriospasm and is not associated with lymphedema. 2. Shiny skin is associated with conditions resulting in arteriospasm and is not associated with lymphedema. 3. In the early stages of lymphedema (Stage 1), reversible swelling is common, especially in the dorsum of the hand. Signs of lymphedema development include slow progression of swelling, which typically has a distal to proximal progression. 4. Nonpitting fibrotic edema is present with Stage 3 lymphedema.

A patient reports pain in the lateral aspect of the proximal forearm. The pain is reproduced with shoulder girdle depression, elbow extension, shoulder medial (internal) rotation, and wrist flexion. The patient's symptoms are relieved when the physical therapist removes the shoulder girdle depression component. Which of the following syndromes is the MOST appropriate clinical interpretation of these findings? 1. Cubital tunnel 2. Pronator teres 3. Radial tunnel 4. Anterior interosseous

3. Radial tunnel The test as described is for the radial nerve. Radial tunnel syndrome involves compression of the deep branch of the radial nerve and presents with symptoms similar to lateral epicondylalgia, such as poorly localized pain over the radial aspect of the proximal forearm. Symptoms are reproduced in the test position described. (p. 764)

A patient who has low back pain is placed in prone position. A physical therapist performs posteroanterior lumbar segmental mobility testing on the patient and provokes tenderness in the L3-L4 segments. The patient is then instructed to stand and bend over the end of the plinth, grab the edges of the plinth in order to stabilize the trunk, and lift both feet off the ground. Which of the following consequent scenarios will be MOST helpful in establishing the patient's prognosis? 1. Inability to secure the upper body on the plinth 2. Inability to lift both lower extremities off the ground 3. Reduced tenderness on repeat palpation with the legs lifted 4. Reduced localized back pain symptoms with the legs lifted

3. Reduced tenderness on repeat palpation with the legs lifted 3. The test being described is the prone instability test. This orthopedic test has a positive result when a patient who has pain upon prone segmental palpation of the lumbar spine demonstrates a reduction of tenderness when asked to stabilize the upper body and lift the legs off the ground. A positive test result implies that the tenderness initially encountered in prone position can be reduced upon activation of the segmental stabilizers of the spine, which contract when the lower extremities are raised off the ground in the second part of the examination. Physical therapists can use this information to determine whether pain can be addressed mainly through lumbar spine stabilization techniques. Patients who have negative results on this test have been found to respond poorly to lumbar stabilization techniques.

With a patient in sitting position, a physical therapist turns the patient's head to one side and then lowers the patient to a supine position while keeping the head extended. Observation of which of the following signs or symptoms would indicate a positive test result? 1. Blurring of vision 2. Rise in blood pressure 3. Rhythmic oscillation of the eye 4. Tingling sensation in the upper extremity

3. Rhythmic oscillation of the eye The Dix-Hallpike maneuver, passive movement of the head from an upright position to the head hanging extended and rotated to 45°, is a standard test performed to establish a diagnosis of benign paroxysmal positional vertigo. When the head is in this position, the physical therapist observes the eyes for evidence of nystagmus. Nystagmus is involuntary, rhythmic oscillation of the eye.

Which of the following instructions is MOST appropriate for teaching a patient with C6 tetraplegia to transfer from a wheelchair to a mat? 1. Keep fingers extended to give a broader base of support. 2. Rotate head and shoulders in the same direction as the desired hip motion. 3. Rotate head and shoulders in the direction opposite to the desired hip motion. 4. Keep both hands next to the knees to lock the elbows.

3. Rotate head and shoulders in the direction opposite to the desired hip motion. 1. The fingers should be kept flexed to preserve tenodesis grip. 2. The head and shoulders should be rotated in the opposite, not same, direction as the desired hip motion. 3. Rotating the head and shoulders in the direction opposite the desired hip movement creates the necessary force to move the lower body in this transfer, given the level of the spinal cord injury. 4. The patient's hands should be kept near the thigh or hips with one hand on the mat and one on the wheelchair.

A patient reports burning medial knee pain that occurs during knee flexion greater than 60°. Resisted knee extension is strong and pain free. The patient denies any popping or instability and there is no history of trauma. There is tenderness to palpation over the anteromedial distal thigh. Which of the following conditions is MOST likely causing the patient's knee pain? 1. Medial meniscal tear 2. Adductor magnus strain 3. Saphenous nerve impingement 4. Medial collateral ligament tear

3. Saphenous nerve impingement 1. As the patient has no history of trauma, popping, or instability, a meniscal tear is unlikely. 2. Adductor strains are typically associated with sports activities. The stem states there is no history of trauma, therefore, this is incorrect. 3. Burning pain is likely neurological in origin. The saphenous nerve has a distribution over the distal anteromedial knee and can be compressed as it exits the adductor canal. Flexion of the knee past 60° can reproduce symptoms of saphenous nerve impingement. 4. As the patient has no history of trauma, popping, or instability, a ligament tear is unlikely.

A patient who sustained a right cerebrovascular accident exhibits a flaccid left arm. During muscle testing, the patient is able to shrug the left shoulder. Which of the following statements MOST accurately explains this response? 1. The anterior lobe of the cerebellum was affected. 2. The vagus nerve (CN X) was not affected. 3. The spinal accessory nerve (CN XI) was not affected. 4. The middle cerebral artery was affected.

3. The spinal accessory nerve (CN XI) was not affected. 3. The upper trapezius, which is involved in shrugging the shoulder, is controlled by the spinal accessory nerve (CN XI). The spinal accessory nerve (CN XI), which is supplied by the corticobulbar tract, was apparently not affected by the stroke and accounts for the patient's ability to shrug a flaccid arm.

A patient with complete C4 tetraplegia is working on a program to increase tolerance to upright position. While the patient is on a tilt table elevated to 60°, the patient begins to have a pounding headache and to exhibit flushing and profuse sweating. The physical therapist should FIRST: 1. lower the tilt table to a flat position. 2. lower the tilt table to 40°. 3. check the patient's urinary catheter. 4. check the patient's blood pressure.

3. check the patient's urinary catheter. 1. Flushing, profuse sweating, and a pounding headache are signs of autonomic dysreflexia. Lowering the patient flat may exacerbate the autonomic dysreflexia symptoms. 2. Flushing, profuse sweating, and a pounding headache are signs of autonomic dysreflexia. Keeping the patient upright is best for managing autonomic dysreflexia. 3. Flushing, profuse sweating, and a pounding headache are signs of autonomic dysreflexia. Among the most common causes is a distended or irritated bladder. The first step is to remove the noxious stimulus. 4. Flushing, profuse sweating, and a pounding headache are signs of autonomic dysreflexia. Checking the patient's blood pressure would be performed after attempts to remove the noxious stimuli.

A patient with Parkinson disease has just been admitted to a rehabilitation unit. The patient is dependent in all transfers and requires moderate assistance of one person to walk 30 ft (9.1 m) with a standard walker. To facilitate good carryover for activities, instruction of the family in transfers should occur: 1. during a home visit after the patient is discharged. 2. just prior to discharging the patient. 3. early in the rehabilitation program. 4. when the family feels ready to take the patient home

3. early in the rehabilitation program. 1. Beginning family training after the patient is discharged is too late in the rehabilitation process. The patient and family will have the most success if training is begun early. 2. Family training should begin earlier in the rehabilitation process. 3. The family should be involved in all stages of planning and treatment. Family involvement can shorten the rehabilitation process and facilitate the patient's return to the community. It is important to have the family involved early in the rehabilitation process rather than wait until the patient is ready to be discharged. 4. Family training should begin earlier in the rehabilitation process.

After spinal joint mobilization procedures, a patient calls the physical therapist and reports a minor dull ache in the treated area of the back that lasted for 2 to 3 hours. Based on this symptom, the therapist should: 1. consider a possible neurological lesion in the area. 2. refer the patient back to the physician. 3. inform the patient that this response is common. 4. add strengthening exercises to the home program.

3. inform the patient that this response is common. 1. There is no indication of neurological involvement with the reported symptoms. 2. There would not be a need to refer to the physician because the patient is describing a normal response to treatment. 3. Joint mobilization procedures may cause some soreness. The therapist should inform the patient of this potential response to treatment. The therapist should re-evaluate the patient and could alter the treatment by waiting an extra day before the next treatment or by decreasing the dosage. 4. The patient's response is normal and the addition of exercises would not alter the response.

A patient who sustained a mild cerebrovascular accident 3 weeks ago is being prepared by a physical therapist for discharge to home and an adult day program. To facilitate the discharge plan, the MOST appropriate health professional for the therapist to consult with is the: 1. skilled nursing coordinator. 2. occupational therapist. 3. medical social worker. 4. primary physician.

3. medical social worker. Medical social service staff deal with home situations and financial supports and act as a resource director on behalf of the patient. A medical social worker would be the most appropriate choice to help arrange and coordinate rehabilitative services for the patient while the patient is at home.

A patient who has a T7 spinal cord injury (ASIA Impairment Scale D) exhibits decreased motor function in the right lower extremity and loss of pain sensation over the left distal trunk and lower extremity. Which of the following syndromes is MOST likely present? 1. Central cord 2. Anterior cord 3. Posterior cord 4. Brown-Séquard

4. Brown-Séquard 1. The patient has decreased motor function on one side and loss of pain sensation several levels below the level of the lesion, on the opposite side. In central cord syndrome, there is damage to the most central aspects of the spinal cord and there is characteristically more severe neurological involvement and impairment of function in the upper extremities than in the lower extremities. Varying degrees of sensory impairment occur but tend to be less severe than motor deficits. (Umphred, p. 462; O'Sullivan, p. 894) 2. The patient has decreased motor function on one side and loss of pain sensation several levels below the level of the lesion, on the opposite side. Anterior cord syndrome or anterior spinal artery syndrome is characterized by loss of motor function and loss of the sense of pain and temperature bilaterally below the level of the lesion (O'Sullivan, p. 894; Umphred, p. 462). 3. The patient has decreased motor function on one side and loss of pain sensation several levels below the level of the lesion, on the opposite side. Posterior cord syndrome is rare, and the clinical picture includes preservation of motor function, sense of pain, and light touch. There is loss of proprioception, stereognosis, two-point discrimination, and vibration below the level of the lesion. (Umphred, p. 462; O'Sullivan, p. 894) 4. The patient has decreased motor function on one side and loss of pain sensation several levels below the level of the lesion, on the opposite side, consistent with Brown-Séquard syndrome. This syndrome is characterized by ipsilateral loss of motor function and position sense and contralateral loss of pain sensation several levels below the lesion. (Umphred p. 462)

A physical therapist is treating a 12-year-old athlete who has had lateral epicondylalgia for 3 weeks. Which of the following modalities presents the GREATEST concern for possible harm to this patient? 1. Ice massage 2. Iontophoresis 3. Moist heat packs 4. Continuous ultrasound

4. Continuous ultrasound 4. Use of therapeutic ultrasound over epiphyseal plates of growing bones is contraindicated. There is potential for damage to the growth plate with intense thermal ultrasound and increased stimulation of bone formation, even with very minimal ultrasound exposure. all options are not contraindicated

Which of the following findings is present in a patient who has Guillain-Barré syndrome, but is not present in a patient who has multiple sclerosis? 1. Clonus 2. Positive Babinski sign 3. Spasticity 4. Hypoactive deep tendon reflexes

4. Hypoactive deep tendon reflexes 4. Guillain-Barré syndrome is a demyelinating disease that affects the lower motor neurons; multiple sclerosis is a demyelinating disease that affects the upper motor neurons. Hypoactive, or absent, deep tendon reflexes are indicative of lower motor neuron damage.

A physical therapist is evaluating a patient who had a right lower lobe resection due to lung cancer 1 day ago. During auscultation of the patient's lungs, the therapist notes low-pitched crackles bilaterally. The patient's vital signs are heart rate of 99 bpm, blood pressure of 115/75 mm Hg, and pulse oximetry reading of 92% while receiving 2 L/minute of oxygen in sitting position at the edge of the bed. Which of the following actions should the therapist take NEXT? 1. Begin walking activities, with the patient receiving 4 L/minute of oxygen. 2. Contact the physician. 3. Perform active range-of-motion exercises with the patient at bedside. 4. Initiate bronchopulmonary hygiene

4. Initiate bronchopulmonary hygiene 1. Although walking usually helps to clear the lungs, the pulse oximetry measure is too low to initiate walking before the patient's lungs are cleared. 2. The therapist should clear the lungs first, not contact the physician. If the therapist is completely unsuccessful with clearing the lungs and the pulse oximetry reading remains low, then contacting the physician may be appropriate. 3. Range of motion exercises are important post thoracotomy, but again, the patient's breathing should be addressed first. 4. The crackles and low pulse oximetry measure indicate that the patient needs better ventilation and secretion clearance due to the recent surgery. Bronchopulmonary hygiene, including postural drainage, percussion, vibration, suctioning, and incentive spirometry, are beneficial in treating and preventing postoperative atelectasis.

Which of the following exercises should increase a patient's shoulder lateral (external) rotation range of motion by contraction of the tight muscle? 1. Isotonic contraction of medial (internal) rotators followed by passive motion into medial (internal) rotation 2. Isometric hold resisting medial (internal) rotation followed by passive motion into medial (internal) rotation 3. Isometric hold resisting lateral (external) rotation followed by passive motion into medial (internal) rotation 4. Isometric hold resisting medial (internal) rotation followed by passive motion into lateral (external) rotation

4. Isometric hold resisting medial (internal) rotation followed by passive motion into lateral (external) rotation 1. Contraction of the medial (internal) rotators and passive motion into medial (internal) rotation would promote medial (internal) rotation range of motion. 2. Passive motion into medial (internal) rotation would not promote lateral (external) rotation range of motion. 3. Isometric hold resisting lateral (external) rotation followed by passive motion into medial (internal) rotation would promote contraction of the shoulder medial (internal) rotators and shoulder medial (internal) rotation range of motion. 4. According to neurophysiological principles, contraction of the involved muscle should cause a reflex relaxation of that muscle. The medial (internal) rotator muscle(s) limit lateral (external) rotation range of motion.

To avoid the appearance of increased motion, what movement MUST be prevented during goniometric measurement of shoulder abduction? 1. Upward rotation of the scapula 2. Medial (internal) rotation of the shoulder 3. Lateral (external) rotation of the shoulder 4. Lateral flexion of the trunk to the opposite side

4. Lateral flexion of the trunk to the opposite side 1. Upward rotation of the scapula would normally accompany the motion of shoulder abduction. 2. Medial (internal) rotation of the shoulder would decrease the available abduction. 3. Lateral (external) rotation of the shoulder would normally accompany the motion of shoulder abduction. 4. During measurement of shoulder abduction, the trunk should be kept straight. If the patient is allowed to laterally flex the trunk, it will give the appearance of increased shoulder abduction, but the motion would be occurring in the spine and not in the shoulder.

A patient demonstrates a right thoracolumbar scoliosis in standing position but no scoliosis in sitting position. Which of the following dysfunctions is MOST likely the cause of the patient's scoliosis? 1. Lumbar facet dysfunction 2. Unilaterally weak gluteus medius 3. Short iliopsoas muscle 4. Leg-length discrepancy

4. Leg-length discrepancy 1. A facet dysfunction would remain in sitting and in standing position. 2. A weak gluteus medius would result in more problems during gait or movement. 3. Although the iliopsoas is passively shortened in sitting position, a short iliopsoas is not a common cause of scoliosis, especially not thoracolumbar scoliosis. 4. Leg length discrepancy is correct because when the patient is sitting, the leg length does not affect the spinal posture; however, a leg length discrepancy could affect posture in standing position.

While a physical therapist is performing transfer training from bed to chair with a patient who had a total knee arthroplasty 2 days ago, the electrocardiograph monitor alarms and the therapist notes that premature ventricular contractions have developed. What is the therapist's BEST course of action at this time? 1. Position the patient on a stable surface and discontinue physical therapy for the day. 2. Continue the transfer to the chair and monitor the patient's oxygen saturation levels. 3. Continue the transfer to the chair and immediately notify the nurse about the premature ventricular contractions. 4. Position the patient on a stable surface and determine the frequency of the premature ventricular contractions.

4. Position the patient on a stable surface and determine the frequency of the premature ventricular contractions. Positioning the patient on a stable surface and determining the frequency of the premature ventricular contractions are the most appropriate actions. The therapist should monitor the electrocardiogram and blood pressure of the patient. Onset of premature ventricular contractions can be benign or stable. Less than six contractions/minute is generally stable, while greater than six contractions/minute is considered less stable. A physical therapist should be able to determine this stability.

Which of the following teaching strategies is MOST appropriate for an older adult patient with mild dementia? 1. Use auditory or visual input separately. 2. Use metaphors to reinforce new concepts. 3. Establish a consistent pace for teaching. 4. Present one piece of new information at a time.

4. Present one piece of new information at a time. 1. Visual and auditory input together can promote learning better than separate visual and auditory input. 2. Concrete examples are easier to understand than metaphors. 3. The pace of learning should be set by the patient. 4. To minimize confusion, the patient should be presented with one new piece of information at a time.

A patient has a history of neck pain that is aggravated by long periods of sitting and becomes progressively worse by evening. Range of motion and strength of the neck and shoulders are within normal limits. Sensation and reflexes are intact in both upper extremities. The patient has a forward head and excessive thoracic kyphosis. Which of the following exercises is MOST appropriate? 1. Deep neck flexor and pectoral stretching 2. Upper trapezius strengthening and levator scapulae stretching 3. Serratus anterior strengthening and rhomboid stretching 4. Rhomboid and deep neck flexor strengthening

4. Rhomboid and deep neck flexor strengthening 1. A chronic forward head is more likely to result in deep neck flexor weakness. Strengthening, not stretching, of the deep neck flexors is most appropriate. 2. A chronic forward head and thoracic kyphosis is most likely to shorten the upper trapezius. Upper trapezius stretching, not strengthening, is most appropriate. 3. Excessive flexion in the upper thoracic spine will put the rhomboids on stretch. Strengthening, not stretching, the rhomboids is most appropriate. 4. The history suggests that prolonged improper positioning of the cervical spine resulted in neck pain. A chronic forward head and kyphosis results in hyperextension in the upper cervical spine and excessive flexion in the upper thoracic spine. Further muscle length adaptation occurs with tight anterior muscles and stretched posterior muscles. Treatment should include correction of muscle weakness or imbalance. Of the options, strengthening of the rhomboids and deep neck flexors is the most appropriate.

Which of the following positions is MOST appropriate to mobilize the radial nerve? 1. Shoulder abduction, elbow extension, and wrist extension 2. Shoulder flexion, elbow flexion, and wrist extension 3. Shoulder abduction, elbow flexion, and wrist flexion 4. Shoulder extension, elbow extension, and wrist flexion

4. Shoulder extension, elbow extension, and wrist flexion 1. The shoulder abduction, elbow extension, and wrist extension position is for mobilization of the median nerve. 2. The shoulder flexion, elbow flexion, and wrist extension position is for mobilization of the ulnar nerve. 3. Shoulder abduction, elbow flexion, and wrist flexion is incorrect because the radial nerve stretch requires elbow extension, not flexion. 4. Shoulder extension, elbow extension, and wrist flexion is the most appropriate position to mobilize the radial nerve.

A physical therapist is preparing for gait training with a young adult patient who has paraplegia. Which of the following gait training options is MOST appropriate for the patient's FIRST session? 1. Swing-through gait pattern with a walker 2. Swing-through gait pattern with forearm crutches 3. Swing-to gait pattern with axillary crutches 4. Swing-to gait pattern in the parallel bars

4. Swing-to gait pattern in the parallel bars 1. A walker is an inappropriate assistive device for a young adult who has paraplegia. 2. Swing-through gait pattern with forearm crutches is the ultimate goal of gait training but would be too difficult for the patient's first attempt. 3. Axillary crutches are not the most efficient gait aide for a patient who has paraplegia. 4. The first-time session of gait training for a patient who has paraplegia should be in the parallel bars. A swing-to gait pattern would be the easiest for the patient to learn initially.

A patient in the 8th month of pregnancy has numbness and tingling of the left hand, except for the little finger (5th digit). She demonstrates edema of the hand and fingers, a positive Tinel sign at the wrist, and a Good (4/5) muscle test grade of the wrist and finger flexors. The MOST appropriate intervention is: 1. a wrist splint to position the wrist in full extension. 2. a hot pack followed by tendon gliding exercises. 3. resistive exercises for the wrist and finger flexors. 4. frequent rest and elevation of the left upper extremity.

4. frequent rest and elevation of the left upper extremity. 1. Compression on the median nerve (carpal tunnel syndrome) is occurring, most likely as a result of swelling associated with the individual being in the 8th month of pregnancy. The wrist should not be positioned in full extension. Initial conservative treatment sometimes includes cock-up splinting to hold the wrist in neutral to 10° of extension, but not full extension. 2. Compression on the median nerve (carpal tunnel syndrome) is occurring, most likely as a result of swelling associated with the individual being in the 8th month of pregnancy. In this case, rest and elevation would do the most to decrease the edema and relieve the symptoms. Although tendon gliding exercises may be used, heat would not be indicated since it may increase the edema. 3. Compression on the median nerve (carpal tunnel syndrome) is occurring, most likely as a result of swelling associated with the individual being in the 8th month of pregnancy. Resistive exercises for the wrist and fingers may aggravate the compression in the carpal tunnel. 4. Compression on the median nerve (carpal tunnel syndrome) is occurring, most likely as a result of swelling associated with the individual being in the 8th month of pregnancy. In this case, rest and elevation would do the most to decrease the edema and relieve the symptoms

During the gait evaluation of a patient, a physical therapist notices that the patient laterally bends excessively toward the right side during the midstance phase on the right. For the therapist to test the suspected muscle for Normal (5/5) strength, the patient should FIRST be positioned: 1. prone with the knee straight. 2. supine with the knee bent. 3. seated with the hip flexed to 110°. 4. sidelying on the left side.

4. sidelying on the left side. 1. Prone with the knee straight would be used to test for hip extensor strength. The most likely cause of laterally bending toward the stance limb is abductor weakness on the stance side. 2. Supine with knee bent is used to test hip flexor weakness below a grade of Fair (3/5). The most likely cause of laterally bending toward the stance limb is abductor weakness on the stance side. 3. Seated hip flexion is used to test the hip flexor muscle strength. The most likely cause of laterally bending toward the stance limb is abductor weakness on the stance side. 4. The most likely cause of laterally bending toward the stance limb is abductor weakness on the stance side. The right abductors would be tested from a left sidelying position for Fair (3/5) or greater strength.


Ensembles d'études connexes

MO-MEGA Middle School Science- JK

View Set

Octroi de crédits - Analyse financière et crédit - HEG-GE banque et finance

View Set

Troubleshooting Digital Television Service Introduction

View Set

First Semester Mix-Tape 🔥 #fire 🔥🔥 !!!!

View Set

Excel- Chapter 12: Creating Charts

View Set

Reading 17. International Trade and Capital Flows

View Set

Cognitive: Chapter 12/13 Music Cog Quiz

View Set

Chapter 36: Management of Patients With Immune Deficiency Disorders - ML4

View Set